Chapter 37: Vascular Disorders, Chapter 37 Vascular Disorders, Chapter 37 Vascular Disorders, Chapter 37: Vascular Disorders, Chapter 37: Vascular Disorders, STROKE--CHAPTER 57, Lewis 57: Stroke, Ch. 57 Stroke, Chapter 57: Stroke, Stroke Ch. 57, Ch....

Pataasin ang iyong marka sa homework at exams ngayon gamit ang Quizwiz!

A patient who has a history of a transient ischemic attack (TIA) has an order for aspirin 160 mg daily. When the nurse is administering medications, the patient says, "I don't need the aspirin today. I don't have a fever." Which action should the nurse take? a. Document that the patient refused the aspirin. b. Tell the patient that the aspirin is used to prevent a fever. c. Explain that the aspirin is ordered to decrease stroke risk. d. Call the health care provider to clarify the medication order.

c. Explain that the aspirin is ordered to decrease stroke risk.

hemorrhagic stroke

occurs when a blood vessel in the brain leaks or ruptures; also known as a bleed

The incidence of ischemic stroke in patients with TIAs and other risk factors is reduced with administration of what medication? a. furosemide b. lovastatin c. daily low dose aspirin d. nimodipine

c. Daily low dose of aspirin

25. A patient with Raynaud disease has a nursing diagnosis of "Ineffective tissue perfusion, related to vasoconstriction" and is being given discharge instructions. What should the nurse include when providing this information? a. Avoid sun exposure. b. Wear gloves and warm socks when outdoors. c. Chafe hands frequently to warm them. d. Wash dishes in warm water.

ANS: Wear gloves and warm socks when outdoors.

22. What medication obtained in a patient's history will lessen the effects of warfarin (Coumadin)? a. Iron supplement for anemia b. Simvastatin (Zocor) for the control of cholesterol c. Furosemide (Lasix) for fluid retention d. Yaz (drospirenone/estradiol) as an oral contraceptive

ANS: Yaz (drospirenone/estradiol) as an oral contraceptive

7. Which technique should the nurse implement when performing the Allen test on a patient to evaluate the adequacy of circulation in the radial artery? a. Asks the patient to relax the hand by the side b. Compresses only the ulnar artery to blanch the hand c. Releases pressure on both arteries at the same time d. observes whether the color is returning to the hand, which indicates perfusion

ANS: observes whether the color is returning to the hand, which indicates perfusion

The nurse is teaching a senior citizen's group about signs and symptoms of a stroke. Which statement provides the group accurate information? "Take the person to the hospital if a headache lasts for more than 24 hours." "Stroke symptoms usually start when the person is awake and physically active." "A person with a transient ischemic attack has mild symptoms that will go away." "Call 911 immediately if a person develops slurred speech or difficulty speaking."

"Call 911 immediately if a person develops slurred speech or difficulty speaking." Medical assistance should be obtained immediately for someone with signs and symptoms of a stroke such as sudden numbness; weakness; paralysis of the face, arm, or leg (especially on one side of the body); sudden confusion; trouble speaking or understanding; slurred speech; sudden trouble seeing in one or both eyes; sudden trouble walking; dizziness; loss of balance or coordination; or a sudden, severe headache with no known cause. A person with signs and symptoms of a transient ischemic attack should seek medical attention immediately because it is unknown if the symptoms will resolve or persist and progress to a stroke. Onset of signs and symptoms of a stroke vary depending on the type. Onset of an ischemic thrombotic stroke usually occurs at rest. Onset of an ischemic embolic stroke is not related to rest or activity, and onset of a hemorrhagic stroke usually occurs with activity.

27. What patient teaching should be included for a patient with varicose veins? a. Weight reduction b. Decreasing exercise c. Wearing a panty girdle d. Standing rather than sitting

ANS: weight reduction

5. A nurse assesses a patient's capillary refill time as less than 3 seconds. What does this assessment indicate? a. Hypertension b. Tissue perfusion c. Excess fluid volume d. Increased blood viscosity

ANS:Tissue perfusion

10. What is a characteristic of a venous stasis ulcer? a. Painlessness b. Poikilothermy c. Pale color d. Location near the groin

ANS:painlessness

What concern should the nurse anticipate for a patient who had a right hemisphere stroke? a. Right-sided hemiplegia b. Speech-language deficits c. Denial of deficits and impulsiveness d. Depression and distress about disability

c. Denial of deficits and impulsiveness

13. A diagnosis of a ruptured cerebral aneurysm has been made in a patient with manifestations of a stroke. The nurse anticipates which treatment option that would be considered for the patient? a. Hyperventilation therapy b. Surgical clipping of the aneurysm c. Administration of hyperosmotic agents d. Administration of thrombolytic therapy

. b. Surgical management with clipping of an aneurysm to decrease rebleeding and vasospasm is an option for a stroke caused by rupture of a cerebral aneurysm. Placement of coils into the lumens of the aneurysm via interventional neuroradiology can also be done to prevent the blood from circulating through the aneurysm and to reduce the risk of rupture. Hyperventilation therapy would increase vasodilation and the potential for hemorrhage. Thrombolytic therapy would be absolutely contraindicated and if a vessel is patent, osmotic diuretics may leak into tissue, pulling fluid out of the vessel and increasing edema.

6. What primarily determines the neurologic functions that are affected by a stroke? a. The amount of tissue area involved b. The rapidity of the onset of symptoms c. The brain area perfused by the affected artery d. The presence or absence of collateral circulation

. c. Clinical manifestations of altered neurologic function differ, depending primarily on the specific cerebral artery involved and the area of the brain that is perfused by the artery. The degree of impairment depends on rapidity of onset, the size of the lesion, and the presence of collateral circulation.

4. The client diagnosed with a right-sided cerebrovascular accident is admitted to the rehabilitation unit. Which interventions should be included in the nursing care plan? Select all that apply. 1. Position the client to prevent shoulder adduction. 2. Turn and reposition the client every shift. 3. Encourage the client to move the affected side. 4. Perform quadriceps exercises three (3) times a day. 5. Instruct the client to hold the fingers in a fist.

1, 3 1. Placing a small pillow under the shoulder will prevent the shoulder from adducting toward the chest and developing a contracture. 2. The client should be repositioned at least every two (2) hours to prevent contractures, pneumonia, skin breakdown, and other complications of immobility. 3. The client should not ignore the paralyzed side, and the nurse must encourage the client to move it as much as possible; a written schedule may assist the client in exercising. 4. These exercises are recommended, but they must be done at least five (5) times a day for 10 minutes to help strengthen the muscles for walking. 5. The fingers are positioned so that they are barely flexed to help prevent contracture of the hand.

3. Which client would the nurse identify as being most at risk for experiencing a CVA? 1. A 55-year-old African American male. 2. An 84-year-old Japanese female. 3. A 67-year-old Caucasian male. 4. A 39-year-old pregnant female.

1. African Americans have twice the rate of CVAs as Caucasians and men have a higher incidence than women; African Americans suffer more extensive damage from a CVA than do people of other cultural groups.

12. A client diagnosed with a subarachnoid hemorrhage has undergone a craniotomy for repair of a ruptured aneurysm. Which intervention will the intensive care nurse implement? 1. Administer a stool softener b.i.d. 2. Encourage the client to cough hourly. 3. Monitor neurological status every shift. 4. Maintain the dopamine drip to keep BP at 160/90.

1. The client is at risk for increased intracranial pressure whenever performing the Valsalva maneuver, which will occur when straining during defecation. Therefore, stool softeners would be appropriate.

7. The client diagnosed with atrial fibrillation has experienced a transient ischemic attack (TIA). Which medication would the nurse anticipate being ordered for the client on discharge? 1. An oral anticoagulant medication. 2. A beta blocker medication. 3. An anti-hyperuricemic medication. 4. A thrombolytic medication.

1. The nurse would anticipate an oral anticoagulant, warfarin (Coumadin), to be prescribed to help prevent thrombi formation in the atria secondary to atrial fibrillation. The thrombi can become embolic and may cause a TIA or CVA (stroke).

You are caring for a patient who is diagnosed with Raynaud's phenomenon. The nurse should plan interventions to address what nursing diagnosis? A) Chronic pain B) Ineffective tissue perfusion C) Impaired skin integrity D) Risk for injury

Ans: Ineffective tissue perfusion Feedback: Raynaud's phenomenon is a form of intermittent arteriolar vasoconstriction resulting in inadequate tissue perfusion. This results in coldness, pain, and pallor of the fingertips or toes. Pain is typically intermittent and acute, not chronic, and skin integrity is rarely at risk. In most cases, the patient is not at a high risk for injury.

9. The client is diagnosed with expressive aphasia. Which psychosocial client problem would the nurse include in the plan of care? 1. Potential for injury. 2. Powerlessness. 3. Disturbed thought processes. 4. Sexual dysfunction

2. Expressive aphasia means that the client cannot communicate thoughts but understands what is being communicated; this leads to frustration, anger, depression, and the inability to verbalize needs, which, in turn, causes the client to have a lack of control and feel powerless.

2. The nurse is assessing a client experiencing motor loss as a result of a left-sided cerebrovascular accident (CVA). Which clinical manifestations would the nurse document? 1. Hemiparesis of the client's left arm and apraxia. 2. Paralysis of the right side of the body and ataxia. 3. Homonymous hemianopsia and diplopia. 4. Impulsive behavior and hostility toward family.

2. The most common motor dysfunction of a CVA is paralysis of one side of the body, hemiplegia; in this case with a left-sided CVA, the paralysis would affect the right side. Ataxia is an impaired ability to coordinate movement.

Which lobe of the cerebrum includes the patient's Broca's speech center? A. Frontal lobe B. Parietal lobe C. Occipital lobe D. Temporal lobe

A. Broca's speech center is located in the frontal lobe and is responsible for the formation of words in speech. The parietal lobe aids in processing of spatial awareness and receiving and processing information about temperature, taste, and touch. The primary visual center is in the occipital lobe. The auditory center for interpreting sound is present in the temporal lobe.

1. A 78-year-old client is admitted to the emergency department with numbness and weakness of the left arm and slurred speech. Which nursing intervention is priority? 1. Prepare to administer recombinant tissue plasminogen activator (rt-PA). 2. Discuss the precipitating factors that caused the symptoms. 3. Schedule for a STAT computed tomography (CT) scan of the head. 4. Notify the speech pathologist for an emergency consult.

3. A CT scan will determine if the client is having a stroke or has a brain tumor or another neurological disorder. If a CVA is diagnosed, the CT scan can determine if it is a hemorrhagic or ischemic accident and guide treatment.

6. The nurse and an unlicensed assistive personnel (UAP) are caring for a client with right-sided paralysis. Which action by the UAP requires the nurse to intervene? 1. The assistant places a gait belt around the client's waist prior to ambulating. 2. The assistant places the client on the back with the client's head to the side. 3. The assistant places a hand under the client's right axilla to move up in bed. 4. The assistant praises the client for attempting to perform ADLs independently.

3. This action is inappropriate and would require intervention by the nurse because pulling on a flaccid shoulder joint could cause shoulder dislocation; the client should be pulled up by placing the arm underneath the back or using a lift sheet.

10. Which assessment data would indicate to the nurse that the client would be at risk for a hemorrhagic stroke? 1. A blood glucose level of 480 mg/dL. 2. A right-sided carotid bruit. 3. A blood pressure of 220/120 mm Hg. 4. The presence of bronchogenic carcinoma.

3. Uncontrolled hypertension is a risk factor for hemorrhagic stroke, which is a ruptured blood vessel inside the cranium.

5. The nurse is planning care for a client experiencing agnosia secondary to a cerebrovascular accident. Which collaborative intervention will be included in the plan of care? 1. Observe the client swallowing for possible aspiration. 2. Position the client in a semi-Fowler's position when sleeping. 3. Place a suction setup at the client's bedside during meals. 4. Refer the client to an occupational therapist for evaluation.

4. A collaborative intervention is an intervention in which another health-care discipline—in this case, occupational therapy—is used in the care of the client.

8. The client has been diagnosed with a cerebrovascular accident (stroke). The client's wife is concerned about her husband's generalized weakness. Which home modification should the nurse suggest to the wife prior to discharge? 1. Obtain a rubber mat to place under the dinner plate. 2. Purchase a long-handled bath sponge for showering. 3. Purchase clothes with Velcro closure devices. 4. Obtain a raised toilet seat for the client's bathroom.

4. Raising the toilet seat is modifying the home and addresses the client's weakness in being able to sit down and get up without straining muscles or requiring lifting assistance from the wife.

11. The 85-year-old client diagnosed with a stroke is complaining of a severe headache. Which intervention should the nurse implement first? 1. Administer a nonnarcotic analgesic. 2. Prepare for STAT magnetic resonance imaging (MRI). 3. Start an intravenous infusion with D5W at 100 mL/hr. 4. Complete a neurological assessment.

4. The nurse must complete a neurological assessment to help determine the cause of the headache before taking any further action.

The nurse in a primary care provider's office is assessing several patients today. Which patient is most at risk for a stroke? A 92-yr-old female patient who takes warfarin for atrial fibrillation A 28-yr-old male patient who uses marijuana after chemotherapy to ease nausea A 72-yr-old male patient who has hypertension and diabetes and smokes tobacco. A 42-yr-old female patient who takes oral contraceptives and has migraine headaches

A 72-yr-old male patient who has hypertension and diabetes and smokes tobacco. Stroke risk increases after 65 years of age. Strokes are more common in men. Hypertension is the single most important modifiable risk factor for stroke. Diabetes is a significant stroke risk factor. Smoking nearly doubles the risk of a stroke. Other risk factors include drug use (especially cocaine), high-dose oral contraception use, migraine headaches, and untreated heart disease, such as atrial fibrillation.

9. Common psychosocial reactions of the stroke patient to the stroke include (select all that apply) a. depression. b. disassociation. c. intellectualization. d. sleep disturbances. e. denial of severity of stroke.

A, D, E

What nursing action is essential when a patient experiences hemianopsia as the result of a left ischemic stroke? A. Place objects within the visual field B. Teach passive range-of-motion exercises C. Instill artificial teardrops into the affected eye D. Reduce time patient is positioned on the left side

A. A stroke in the left hemisphere will lead to a loss of the right visual field of each eye; objects should be placed within the patient's view. Passive ROM exercises, artificial teardrops, and reducing time patient is positioned on the left side are not related to hemianopsia (loss of vision in half of visual field of one or both eyes).

The family members of a patient with the diagnosis of cerebrovascular accident (CVA, also known as "brain attack") express concern that the patient often becomes uncontrollably tearful during their visits. What should the nurse include in a response? A. Emotional lability is associated with brain trauma B. Their presence allows the patient to express feelings C. The patient is depressed about the loss of functional abilities D. Nonverbal expressions of feelings are more accurate than verbal ones

A. Emotional lability is associated with brain trauma from ischemia or injury. The frontal lobe, hypothalamus, thalamus, and cortical limbic system are involved in expression of emotions. Emotional lability is not limited to interactions with family. Although the patient may be depressed, the uncontrollable tearfulness is because of the disease process. Although nonverbal messages are often helpful in determining emotional response, these emotional outbursts may be unrelated to feelings.

A patient is diagnosed with a brain attack (cerebrovascular accident [CVA]). The baseline vital signs are a pulse rate of 78 bpm and a BP of 120/80 mm Hg. The nurse continues to monitor the vital signs and recognizes that which changes in vital signs indicate increased intracranial pressure (ICP)? A. Pulse 50 bpm and BP 140/60 mm Hg B. Pulse 56 bpm and BP 130/110 mm Hg C. Pulse 60 bpm and BP 126/96 mm Hg D. Pulse 120 bpm and BP 80/60 mm Hg

A. Increased intracranial pressure is evidenced by widening of pulse pressure and a decreased pulse rate. Pulse 56 bpm and BP 130/110 mm Hg, pulse 60 bpm and BP 126/96 mm Hg, and pulse 120 bpm and BP 80/60 mm Hg do not meet these criteria.

A patient had a cerebrovascular accident (also known as a "brain attack"), and bed rest is prescribed. What can the nurse use to best prevent footdrop in this patient? A. Splint B. Block C. Cradles D. Sandbags

A. Various types of splints or boots are available to keep the foot in a position of functional alignment. Blocks elevate the frame of the bed and have no effect on the position of the feet. Although a cradle will keep the pressure of the linen off the patient's feet, which otherwise may promote footdrop, the cradle does not maintain functional alignment of the ankle. Sandbags help prevent rotation of an extremity or the head; they are not used to prevent footdrop.

30. A 63-year-old patient who began experiencing right arm and leg weakness is admitted to the emergency department. In which order will the nurse implement these actions included in the stroke protocol? (Put a comma and a space between each answer choice [A, B, C, D].) a. Obtain computed tomography (CT) scan without contrast. b. Infuse tissue plasminogen activator (tPA). c. Administer oxygen to keep O2 saturation >95%. d. Use National Institute of Health Stroke Scale to assess patient.

ANS: C, D, A, B The initial actions should be those that help with airway, breathing, and circulation. Baseline neurologic assessments should be done next. A CT scan will be needed to rule out hemorrhagic stroke before tPA can be administered. DIF: Cognitive Level: Apply (application) REF: 1401 | 1404 | 1398 OBJ: Special Questions: Prioritization TOP: Nursing Process: Implementation MSC: NCLEX: Physiological Integrity

When assessing a patient with possible peripheral artery disease (PAD) the nurse obtains a brachial blood pressure (BP) of 147/82 mm Hg and an ankle pressure of 112/74 mm Hg. The nurse calculates the patient's ankle-brachial index (ABI) as ________ (round up to the nearest hundredth).

ANS: 0.76 The ABI is calculated by dividing the ankle systolic BP by the brachial systolic BP. DIF: Cognitive Level: Apply (application) REF: 805 TOP: Nursing Process

29. A nurse suspects a circulatory disorder in one leg. Which assessments should the nurse include when comparing both legs? (Select all that apply.) a. Color b. Warmth c. Muscle strength d. Pulse quality e. Hair loss on extremity

ANS: a. Color b. Warmth d.pulse quality e.hair loss on extremity

28. Why do older persons adapt more slowly to changes in the peripheral vascular system? (Select all that apply.) a. Slowing heart rate b. Decreasing cardiac output c. Increasing stroke volume d.stiffening of blood vessels e. thickening of aorta

ANS: a. Slowing heart rate b. Decreasing cardiac output d. stiffening of blood vessels e.thickening of aorta

31. A nurse explains that when a patient history reveals a recent episode of vomiting and diarrhea, the nurse anticipates that this fluid loss will cause _____ and increased blood viscosity.

ANS: hemoconcentration

30. A nurse explains that the lining of a vessel that allows for smooth blood flow and also reduced resistance in the vessel is the _____ of the vessel.

ANS: intima

21. A patient has returned from a vein ligation and stripping. What are the appropriate instructions for a nurse to provide? a. Dangle the legs to prevent edema. b. Cross the legs to apply pressure. c. Wear compression stockings to promote circulation. d. Remove the drain after 24 hours.

ANS: wear compression stockings to promote circulation

20. Which statement made by a patient indicates to the nurse that a teaching plan for the use of warfarin was not effective? a. "I don't take aspirin anymore." b. "I read that grapefruit interferes with warfarin." c. "I'm drinking too much tea. My urine looks like tea." d. "I wear my medical alert bracelet all the time."

ANS: "im drinking too much tea. My urine looks like tea."

When assessing a patient with possible peripheral artery disease (PAD), the nurse obtains a brachial blood pressure (BP) of 147/82 mm Hg and an ankle pressure of 112/74 mm Hg. The nurse calculates the patient's ankle-brachial index (ABI) as ________ (round up to the nearest hundredth).

ANS: 0.76 The ABI is calculated by dividing the ankle systolic BP by the brachial systolic BP.

15. A nurse cautions a patient with peripheral vascular disease (PVD) that continued smoking causes detrimental vasoconstriction for up to ____ after only one cigarette. a. 10 minutes b. 20 minutes c. 30 minutes d. 1 hour

ANS: 1 hr

The nurse is developing a discharge teaching plan for a patient diagnosed with thromboangiitis obliterans (Buerger's disease). Which expected outcome has the highest priority for this patient? a. Cessation of all tobacco use b. Control of serum lipid levels c. Maintenance of appropriate weight d. Demonstration of meticulous foot care

ANS: A Absolute cessation of nicotine use is needed to reduce the risk for amputation in patients with Buerger's disease. Other therapies have limited success in treatment of this disease. DIF: Cognitive Level: Analyze (analysis) REF: 809 OBJ: Special Questions: Prioritization TOP: Nursing Process: Planning MSC: NCLEX: Physiological Integrity

Which patient statement to the nurse is most consistent with the diagnosis of venous insufficiency? a. "I can't get my shoes on at the end of the day." b. "I can't ever seem to get my feet warm enough." c. "I have burning leg pains after I walk two blocks." d. "I wake up during the night because my legs hurt."

ANS: A Because the edema associated with venous insufficiency increases when the patient has been standing, shoes will feel tighter at the end of the day. The other patient statements are characteristic of peripheral artery disease. DIF: Cognitive Level: Apply (application) REF: 826 TOP: Nursing Process: Assessment MSC: NCLEX: Physiological Integrity

An older patient with a history of an abdominal aortic aneurysm arrives at the emergency department (ED) with severe back pain and absent pedal pulses. Which action should the nurse take first? a. Check the blood pressure. b. Draw blood for laboratory testing. c. Assess for the presence of an abdominal bruit. d. Determine any family history of heart disease.

ANS: A Because the patient appears to be experiencing aortic dissection, the nurse's first action should be to determine the hemodynamic status by assessing blood pressure. The other actions may also be done, but they will not provide information to determine what interventions are needed immediately. DIF: Cognitive Level: Analyze (analysis) REF: 814 OBJ: Special Questions: Prioritization TOP: Nursing Process: Implementation MSC: NCLEX: Physiological Integrity

When evaluating the discharge teaching for a patient with chronic peripheral artery disease (PAD) the nurse determines a need for further instruction when the patient says "I will a. use a heating pad on my feet at night to increase the circulation." b. buy some loose clothes that do not bind across my legs or waist." c. walk to the point of pain rest, and walk again for at least 30 minutes 3 times a week." d. change my position every hour and avoid long periods of sitting with my legs crossed.",

ANS: A Because the patient has impaired circulation and sensation to the feet, the use of a heating pad could lead to burns. The other patient statements are correct and indicate that teaching has been successful. DIF: Cognitive Level: Apply (application) REF: 804 TOP: Nursing Process: Evaluation MSC: NCLEX: Physiological Integrity

The nurse is caring for a patient with critical limb ischemia who has just arrived on the nursing unit after having percutaneous transluminal balloon angioplasty. Which action should the nurse perform first? a. Obtain vital signs. b. Teach wound care. c. Assess pedal pulses. d. Check the wound site.

ANS: A Bleeding is a possible complication after catheterization of the femoral artery, so the nurse's first action should be to assess for changes in vital signs that might indicate hemorrhage. The other actions are also appropriate but can be done after determining that bleeding is not occurring. DIF: Cognitive Level: Analyze (analysis) REF: 804 OBJ: Special Questions: Prioritization TOP: Nursing Process: Implementation MSC: NCLEX: Physiological Integrity

9. A female patient who had a stroke 24 hours ago has expressive aphasia. An appropriate nursing intervention to help the patient communicate is to a. ask questions that the patient can answer with "yes" or "no." b. develop a list of words that the patient can read and practice reciting. c. have the patient practice her facial and tongue exercises with a mirror. d. prevent embarrassing the patient by answering for her if she does not respond.

ANS: A Communication will be facilitated and less frustrating to the patient when questions that require a "yes" or "no" response are used. When the language areas of the brain are injured, the patient might not be able to read or recite words, which will frustrate the patient without improving communication. Expressive aphasia is caused by damage to the language areas of the brain, not by the areas that control the motor aspects of speech. The nurse should allow time for the patient to respond. DIF: Cognitive Level: Apply (application) REF: 1361 TOP: Nursing Process: Implementation MSC: NCLEX: Physiological Integrity

6. A patient with carotid atherosclerosis asks the nurse to describe a carotid endarterectomy. Which response by the nurse is accurate? a. "The obstructing plaque is surgically removed from an artery in the neck." b. "The diseased portion of the artery in the brain is replaced with a synthetic graft." c. "A wire is threaded through an artery in the leg to the clots in the carotid artery and the clots are removed." d. "A catheter with a deflated balloon is positioned at the narrow area, and the balloon is inflated to flatten the plaque."

ANS: A In a carotid endarterectomy, the carotid artery is incised and the plaque is removed. The response beginning, "The diseased portion of the artery in the brain is replaced" describes an arterial graft procedure. The answer beginning, "A catheter with a deflated balloon is positioned at the narrow area" describes an angioplasty. The final response beginning, "A wire is threaded through the artery" describes the mechanical embolus removal in cerebral ischemia (MERCI) procedure. DIF: Cognitive Level: Understand (comprehension) REF: 1397 TOP: Nursing Process: Implementation MSC: NCLEX: Physiological Integrity

The nurse is caring for a patient immediately after repair of an abdominal aortic aneurysm. On assessment the patient has absent popliteal posterior tibial, and dorsalis pedis pulses. The legs are cool and mottled. Which action should the nurse take first? a. Notify the surgeon and anesthesiologist. b. Wrap both the legs in a warming blanket. c. Document the findings and recheck in 15 minutes. d. Compare findings to the preoperative assessment of the pulses.

ANS: A Lower extremity pulses may be absent for a short time after surgery because of vasospasm and hypothermia. Decreased or absent pulses together with a cool and mottled extremity may indicate embolization or graft occlusion. These findings should be reported to the surgeon immediately because this is an emergency situation. Because pulses are marked before surgery, the nurse would know whether pulses were present before surgery before notifying the health care providers about the absent pulses. Because the patient's symptoms may indicate graft occlusion or multiple emboli and a possible need to return to surgery, it is not appropriate to wait 15 minutes before taking action. A warming blanket will not improve the circulation to the patient's legs. DIF: Cognitive Level: Analyze (analysis) REF: 814 OBJ: Special Questions: Prioritization TOP: Nursing Process: Implementation MSC: NCLEX: Physiological Integrity

After teaching a patient with newly diagnosed Raynaud's phenomenon about how to manage the condition which action by the patient best demonstrates that the teaching has been effective? a. The patient exercises indoors during the winter months. b. The patient immerses hands in hot water when they turn pale. c. The patient takes pseudoephedrine (Sudafed) for cold symptoms. d. The patient avoids taking nonsteroidal antiinflammatory drugs (NSAIDs).

ANS: A Patients should avoid temperature extremes by exercising indoors when it is cold. To avoid burn injuries, the patient should use warm rather than hot water to warm the hands. Pseudoephedrine is a vasoconstrictor and should be avoided. There is no reason to avoid taking NSAIDs with Raynaud's phenomenon. DIF: Cognitive Level: Apply (application) REF: 809 TOP: Nursing Process: Evaluation MSC: NCLEX: Physiological Integrity

A patient in the outpatient clinic has a new diagnosis of peripheral artery disease (PAD). Which group of drugs will the nurse plan to include when teaching about PAD management? a. Statins b. Antibiotics c. Thrombolytics d. Anticoagulants

ANS: A Research indicates that statin use by patients with PAD improves multiple outcomes. There is no research that supports the use of the other drug categories in PAD. DIF: Cognitive Level: Apply (application) REF: 805 TOP: Nursing Process: Planning MSC: NCLEX: Physiological Integrity

Which nursing action should be included in the plan of care after endovascular repair of an abdominal aortic aneurysm? a. Record hourly chest tube drainage. b. Monitor fluid intake and urine output. c. Assess the abdominal incision for redness. d. Teach the patient to plan for a long recovery period.

ANS: B Because renal artery occlusion can occur after endovascular repair, the nurse should monitor parameters of renal function such as intake and output. Chest tubes will not be needed for endovascular surgery, the recovery period will be short, and there will not be an abdominal wound. DIF: Cognitive Level: Apply (application) REF: 815 TOP: Nursing Process: Planning MSC: NCLEX: Physiological Integrity

A patient who is 2 days post femoral popliteal bypass graft to the right leg is being cared for on the vascular unit. Which action by a licensed practical/vocational nurse (LPN/LVN) caring for the patient requires the registered nurse (RN) to intervene? a. The LPN/LVN has the patient to sit in a chair for 2 hours. b. The LPN/LVN gives the prescribed aspirin after breakfast. c. The LPN/LVN assists the patient to walk 40 feet in the hallway. d. The LPN/LVN places the patient in Fowler's position for meals.

ANS: A The patient should avoid sitting for long periods because of the increased stress on the suture line caused by leg edema and because of the risk for venous thromboembolism (VTE). The other actions by the LPN/LVN are appropriate. DIF: Cognitive Level: Apply (application) REF: 806 OBJ: Special Questions: Delegation TOP: Nursing Process: Implementation MSC: NCLEX: Safe and Effective Care Environment

13. A 40-year-old patient has a ruptured cerebral aneurysm and subarachnoid hemorrhage. Which intervention will be included in the care plan? a. Apply intermittent pneumatic compression stockings. b. Assist to dangle on edge of bed and assess for dizziness. c. Encourage patient to cough and deep breathe every 4 hours. d. Insert an oropharyngeal airway to prevent airway obstruction.

ANS: A The patient with a subarachnoid hemorrhage usually has minimal activity to prevent cerebral vasospasm or further bleeding and is at risk for venous thromboembolism (VTE). Activities such as coughing and sitting up that might increase intracranial pressure (ICP) or decrease cerebral blood flow are avoided. Because there is no indication that the patient is unconscious, an oropharyngeal airway is inappropriate. DIF: Cognitive Level: Apply (application) REF: 1359 TOP: Nursing Process: Planning MSC: NCLEX: Physiological Integrity

10. For a patient who had a right hemisphere stroke the nurse establishes a nursing diagnosis of a. risk for injury related to denial of deficits and impulsiveness. b. impaired physical mobility related to right-sided hemiplegia. c. impaired verbal communication related to speech-language deficits. d. ineffective coping related to depression and distress about disability.

ANS: A The patient with right-sided brain damage typically denies any deficits and has poor impulse control, leading to risk for injury when the patient attempts activities such as transferring from a bed to a chair. Right-sided brain damage causes left hemiplegia. Left-sided brain damage typically causes language deficits. Left-sided brain damage is associated with depression and distress about the disability. DIF: Cognitive Level: Apply (application) REF: 1350 TOP: Nursing Process: Diagnosis MSC: NCLEX: Physiological Integrity

26. After receiving change-of-shift report on the following four patients, which patient should the nurse see first? a. A 60-year-old patient with right-sided weakness who has an infusion of tPA prescribed b. A 50-year-old patient who has atrial fibrillation and a new order for warfarin (Coumadin) c. A 40-year-old patient who experienced a transient ischemic attack yesterday who has a dose of aspirin due d. A 30-year-old patient with a subarachnoid hemorrhage 2 days ago who has nimodipine (Nimotop) scheduled

ANS: A tPA needs to be infused within the first few hours after stroke symptoms start in order to be effective in minimizing brain injury. The other medications should also be given as quickly as possible, but timing of the medications is not as critical. DIF: Cognitive Level: Apply (application) REF: 1398 OBJ: Special Questions: Prioritization; Multiple Patients TOP: Nursing Process: Implementation MSC: NCLEX: Safe and Effective Care Environment

Which patient statement to the nurse is most consistent with the diagnosis of venous insufficiency? a. "I can't get my shoes on at the end of the day." b. "I can't ever seem to get my feet warm enough." c. "I have burning leg pains after I walk two blocks." d. "I wake up during the night because my legs hurt."

ANS: A Because the edema associated with venous insufficiency increases when the patient has been standing, shoes will feel tighter at the end of the day. The other patient statements are characteristic of peripheral artery disease.

When evaluating the discharge teaching for a patient with chronic peripheral artery disease (PAD), the nurse determines a need for further instruction when the patient says, "I will a. use a heating pad on my feet at night to increase the circulation." b. buy some loose clothes that do not bind across my legs or waist." c. walk to the point of pain, rest, and walk again for at least 30 minutes 3 times a week." d. change my position every hour and avoid long periods of sitting with my legs crossed."

ANS: A Because the patient has impaired circulation and sensation to the feet, the use of a heating pad could lead to burns. The other patient statements are correct and indicate that teaching has been successful.

The nurse is caring for a patient with critical limb ischemia who has just arrived on the nursing unit after having percutaneous transluminal balloon angioplasty. Which action should the nurse perform first? a. Obtain vital signs. c. Assess pedal pulses. b. Teach wound care. d. Check the wound site.

ANS: A Bleeding is a possible complication after catheterization of the femoral artery, so the nurse's first action should be to assess for changes in vital signs that might indicate hemorrhage. The other actions are also appropriate but can be done after determining that bleeding is not occurring.

A patient in the outpatient clinic has a new diagnosis of peripheral artery disease (PAD). Which group of drugs will the nurse plan to include when teaching about PAD management? a. Statins b. Antibiotics c. Thrombolytics d. Anticoagulants

ANS: A Research indicates that statin use by patients with PAD improves multiple outcomes. There is no research that supports the use of the other drug categories in PAD.

23. An obese postsurgical patient complains of sudden discomfort in her leg. The nurse assesses the leg and finds it cold and pale with no pedal or popliteal pulse. What should the nurse suspect? a. Venous thrombosis b. Arterial occlusion c. Vascular spasm d. Paresthesia

ANS: Arterial occlusion

9. A nurse notes ulcerations on the surfaces of a patient's toes. What should this assessment most likely indicate? a. Skin breakdown from pressure b. Nutritional deficit c. Venous stasis d. Arterial stasis

ANS: Arterial stasis

The nurse is admitting a patient newly diagnosed with peripheral artery disease. Which admission order should the nurse question? a. Cilostazol drug therapy b. Omeprazole drug therapy c. Use of treadmill for exercise d. Exercise to the point of discomfort

ANS: B Because the antiplatelet effect of clopidogrel is reduced when it is used with omeprazole, the nurse should clarify this order with the health care provider. The other interventions are appropriate for a patient with peripheral artery disease. DIF: Cognitive Level: Apply (application) REF: 805 TOP: Nursing Process: Assessment MSC: NCLEX: Physiological Integrity

5. When teaching about clopidogrel (Plavix), the nurse will tell the patient with cerebral atherosclerosis: a. to monitor and record the blood pressure daily. b. to call the health care provider if stools are tarry. c. that clopidogrel will dissolve clots in the cerebral arteries. d. that clopidogrel will reduce cerebral artery plaque formation.

ANS: B Clopidogrel inhibits platelet function and increases the risk for gastrointestinal bleeding, so patients should be advised to notify the health care provider about any signs of bleeding. The medication does not lower blood pressure, decrease plaque formation, or dissolve clots. DIF: Cognitive Level: Apply (application) REF: 1353 TOP: Nursing Process: Implementation MSC: NCLEX: Physiological Integrity

While working in the outpatient clinic the nurse notes that a patient has a history of intermittent claudication. Which statement by the patient would support this information? a. "When I stand too long, my feet start to swell." b. "My legs cramp when I walk more than a block." c. "I get short of breath when I climb a lot of stairs." d. "My fingers hurt when I go outside in cold weather."

ANS: B Cramping that is precipitated by a consistent level of exercise is descriptive of intermittent claudication. Finger pain associated with cold weather is typical of Raynaud's phenomenon. Shortness of breath that occurs with exercise is not typical of intermittent claudication, which is reproducible. Swelling associated with prolonged standing is typical of venous disease. DIF: Cognitive Level: Apply (application) REF: 803 TOP: Nursing Process: Assessment MSC: NCLEX: Physiological Integrity

16. Several weeks after a stroke, a 50-year-old male patient has impaired awareness of bladder fullness, resulting in urinary incontinence. Which nursing intervention will be best to include in the initial plan for an effective bladder training program? a. Limit fluid intake to 1200 mL daily to reduce urine volume. b. Assist the patient onto the bedside commode every 2 hours. c. Perform intermittent catheterization after each voiding to check for residual urine. d. Use an external "condom" catheter to protect the skin and prevent embarrassment.

ANS: B Developing a regular voiding schedule will prevent incontinence and may increase patient awareness of a full bladder. A 1200 mL fluid restriction may lead to dehydration. Intermittent catheterization and use of a condom catheter are appropriate in the acute phase of stroke, but should not be considered solutions for long-term management because of the risks for urinary tract infection (UTI) and skin breakdown. DIF: Cognitive Level: Apply (application) REF: 1406 TOP: Nursing Process: Planning MSC: NCLEX: Physiological Integrity

A patient has a 6-cm thoracic aortic aneurysm that was discovered during routine chest x-ray. When obtaining an admission history from the patient it will be most important for the nurse to ask about a. low back pain. c. abdominal tenderness. b. trouble swallowing. d. changes in bowel habits.

ANS: B Difficulty swallowing may occur with a thoracic aneurysm because of pressure on the esophagus. The other symptoms will be important to assess for in patients with abdominal aortic aneurysms. DIF: Cognitive Level: Analyze (analysis) REF: 810 TOP: Nursing Process: Assessment MSC: NCLEX: Physiological Integrity

The health care provider prescribes an infusion of heparin and daily partial thromboplastin time (PTT) testing for a patient with venous thromboembolism (VTE). The nurse will plan to a. decrease the infusion when the PTT value is 65 seconds. b. avoid giving IM medications to prevent localized bleeding. c. have vitamin K available in case reversal of the heparin is needed. d. monitor posterior tibial and dorsalis pedis pulses with the Doppler.

ANS: B Intramuscular injections are avoided in patients receiving anticoagulation to prevent hematoma formation and bleeding from the site. A PTT of 65 seconds is within the therapeutic range. Vitamin K is used to reverse warfarin. Pulse quality is not affected by VTE. DIF: Cognitive Level: Apply (application) REF: 823 TOP: Nursing Process: Planning MSC: NCLEX: Physiological Integrity

The nurse who works in the vascular clinic has several patients with venous insufficiency scheduled today. Which patient should the nurse assign to an experienced licensed practical/vocational nurse (LPN/LVN)? a. Patient who has been complaining of increased edema and skin changes in the legs b. Patient who needs wound care for a chronic venous stasis ulcer on the right lower leg c. Patient who has a history of venous thromboembolism and is complaining of dyspnea d. Patient who needs teaching about elastic compression stockings for venous insufficiency

ANS: B LPN education and scope of practice includes wound care. The other patients, which require more complex assessments or education, should be managed by the RN. DIF: Cognitive Level: Apply (application) REF: 827 OBJ: Special Questions: Delegation | Special Questions: Multiple Patients TOP: Nursing Process: Planning MSC: NCLEX: Safe and Effective Care Environment

A patient with a venous thromboembolism (VTE) is started on enoxaparin (Lovenox) and warfarin (Coumadin). The patient asks the nurse why two medications are necessary. Which response by the nurse is most accurate? a. "Taking two blood thinners greatly reduces the risk for another clot to form." b. "Enoxaparin will work right away but warfarin takes several days to begin preventing clots." c. "Enoxaparin will start to dissolve the clot, and warfarin will prevent any more clots from forming." d. "Because of the risk for a blood clot in the lungs, it is important for you to take more than one blood thinner."

ANS: B Low molecular weight heparin (LMWH) is used because of the immediate effect on coagulation and discontinued once the international normalized ratio (INR) value indicates that the warfarin has reached a therapeutic level. LMWH has no thrombolytic properties. The use of two anticoagulants is not related to the risk for pulmonary embolism, and two are not necessary to reduce the risk for another VTE. Anticoagulants do not thin the blood. DIF: Cognitive Level: Apply (application) REF: 820 TOP: Nursing Process: Implementation MSC: NCLEX: Physiological Integrity

The nurse has started discharge teaching for a patient who is to continue warfarin (Coumadin) after hospitalization for venous thromboembolism (VTE). The nurse determines that additional teaching is needed when the patient says which of the following? a. "I should get a Medic Alert device stating that I take warfarin." b. "I should reduce the amount of green leafy vegetables that I eat." c. "I will need routine blood tests to monitor the effects of the warfarin." d. "I will check with my health care provider before I begin any new drugs."

ANS: B Patients taking warfarin are taught to follow a consistent diet with regard to foods that are high in vitamin K, such as green, leafy vegetables. The other patient statements are accurate. DIF: Cognitive Level: Apply (application) REF: 820 TOP: Nursing Process: Evaluation MSC: NCLEX: Physiological Integrity

2. The most common cause of cerebrovascular disease is: a. arteriosclerosis. b. embolism. c. hypertensive changes. d. vasospasm.

a

27. The nurse is caring for a patient who has just returned after having left carotid artery angioplasty and stenting. Which assessment information is of most concern to the nurse? a. The pulse rate is 102 beats/min. b. The patient has difficulty speaking. c. The blood pressure is 144/86 mm Hg. d. There are fine crackles at the lung bases.

ANS: B Small emboli can occur during carotid artery angioplasty and stenting, and the aphasia indicates a possible stroke during the procedure. Slightly elevated pulse rate and blood pressure are not unusual because of anxiety associated with the procedure. Fine crackles at the lung bases may indicate atelectasis caused by immobility during the procedure. The nurse should have the patient take some deep breaths. DIF: Cognitive Level: Apply (application) REF: 1394 OBJ: Special Questions: Prioritization TOP: Nursing Process: Assessment MSC: NCLEX: Physiological Integrity

Which instructions should the nurse include in a teaching plan for an older patient newly diagnosed with peripheral artery disease (PAD)? a. "Exercise only if you do not experience any pain." b. "It is very important that you stop smoking cigarettes." c. "Try to keep your legs elevated whenever you are sitting." d. "Put elastic compression stockings on early in the morning."

ANS: B Smoking cessation is essential for slowing the progression of PAD to critical limb ischemia and reducing the risk of myocardial infarction and death. Circulation to the legs will decrease if the legs are elevated. Patients with PAD are taught to exercise to the point of feeling pain, rest, and then resume walking. Support hose are not used for patients with PAD. DIF: Cognitive Level: Apply (application) REF: 817 TOP: Nursing Process: Planning MSC: NCLEX: Physiological Integrity

After receiving change of shift report which patient admitted to the emergency department should the nurse assess first? a. A 67-yr-old patient who has a gangrenous left foot ulcer with a weak pedal pulse b. A 50-yr-old patient who is complaining of sudden sharp and severe upper back pain c. A 39-yr-old patient who has right calf tenderness, redness, and swelling after a plane ride d. A 58-yr-old patient who is taking anticoagulants for atrial fibrillation and has black stools

ANS: B The patient's presentation of sudden sharp and severe upper back pain is consistent with dissecting thoracic aneurysm, which will require the most rapid intervention. The other patients also require rapid intervention but not before the patient with severe pain. DIF: Cognitive Level: Analyze (analysis) REF: 810 OBJ: Special Questions: Prioritization | Special Questions: Multiple Patients TOP: Nursing Process: Assessment MSC: NCLEX: Physiological Integrity

24. Which information about the patient who has had a subarachnoid hemorrhage is most important to communicate to the health care provider? a. The patient complains of having a stiff neck. b. The patient's blood pressure (BP) is 90/50 mm Hg. c. The patient reports a severe and unrelenting headache. d. The cerebrospinal fluid (CSF) report shows red blood cells (RBCs).

ANS: B To prevent cerebral vasospasm and maintain cerebral perfusion, blood pressure needs to be maintained at a level higher than 90 mm Hg systolic after a subarachnoid hemorrhage. A low BP or drop in BP indicates a need to administer fluids and/or vasopressors to increase the BP. An ongoing headache, RBCs in the CSF, and a stiff neck are all typical clinical manifestations of a subarachnoid hemorrhage and do not need to be rapidly communicated to the health care provider. DIF: Cognitive Level: Apply (application) REF: 1350 OBJ: Special Questions: Prioritization TOP: Nursing Process: Assessment MSC: NCLEX: Physiological Integrity

Which actions could the nurse delegate to unlicensed assistive personnel (UAP) who are providing care for a patient who is at risk for venous thromboembolism? a. Monitor for any bleeding after anticoagulation therapy is started. b. Apply sequential compression device whenever the patient is in bed. c. Ask the patient about use of herbal medicines or dietary supplements. d. Instruct the patient to call immediately if any shortness of breath occurs.

ANS: B UAP training includes the use of equipment that requires minimal nursing judgment, such as sequential compression devices. Patient assessment and teaching require more education and critical thinking and should be done by the registered nurse (RN). DIF: Cognitive Level: Apply (application) REF: 824 OBJ: Special Questions: Delegation TOP: Nursing Process: Planning MSC: NCLEX: Safe and Effective Care Environment

18. A patient in the clinic reports a recent episode of dysphasia and left-sided weakness at home that resolved after 2 hours. The nurse will anticipate teaching the patient about a. tPA b. aspirin c. warfarin (Coumadin) d. nimodipine

ANS: B After a transient ischemic attack, patients typically are started on medications such as aspirin to inhibit platelet function and decrease stroke risk. tPA is used for acute ischemic stroke. Coumadin is usually used for patients with atrial fibrillation. Nimodipine is used to prevent cerebral vasospasm after a subarachnoid hemorrhage. DIF: Cognitive Level: Apply (application) REF: 1353 TOP: Nursing Process: Planning MSC: NCLEX: Physiological Integrity

The nurse is admitting a patient newly diagnosed with peripheral artery disease. Which admission order should the nurse question? a. Cilostazol drug therapy b. Omeprazole drug therapy c. Use of treadmill for exercise d. Exercise to the point of discomfort

ANS: B Because the antiplatelet effect of clopidogrel is reduced when it is used with omeprazole, the nurse should clarify this order with the health care provider. The other interventions are appropriate for a patient with peripheral artery disease.

While working in the outpatient clinic, the nurse notes that a patient has a history of intermittent claudication. Which statement by the patient would support this information? a. "When I stand too long, my feet start to swell." b. "My legs cramp when I walk more than a block." c. "I get short of breath when I climb a lot of stairs." d. "My fingers hurt when I go outside in cold weather."

ANS: B Cramping that is precipitated by a consistent level of exercise is descriptive of intermittent claudication. Finger pain associated with cold weather is typical of Raynaud's phenomenon. Shortness of breath that occurs with exercise is not typical of intermittent claudication, which is reproducible. Swelling associated with prolonged standing is typical of venous disease.

The health care provider prescribes an infusion of heparin and daily partial thromboplastin time (PTT) testing for a patient with venous thromboembolism (VTE). The nurse will plan to a. decrease the infusion when the PTT value is 65 seconds. b. avoid giving IM medications to prevent localized bleeding. c. have vitamin K available in case reversal of the heparin is needed. d. monitor posterior tibial and dorsalis pedis pulses with the Doppler

ANS: B Intramuscular injections are avoided in patients receiving anticoagulation to prevent hematoma formation and bleeding from the site. A PTT of 65 seconds is within the therapeutic range. Vitamin K is used to reverse warfarin. Pulse quality is not affected by VTE.

The nurse who works in the vascular clinic has several patients with venous insufficiency scheduled today. Which patient should the nurse assign to an experienced licensed practical/vocational nurse (LPN/LVN)? a. Patient who has been complaining of increased edema and skin changes in the legs b. Patient who needs wound care for a chronic venous stasis ulcer on the right lower leg c. Patient who has a history of venous thromboembolism and is complaining of dyspnea d. Patient who needs teaching about elastic compression stockings for venous insufficiency

ANS: B LPN education and scope of practice includes wound care. The other patients, which require more complex assessments or education, should be managed by the RN

A patient with a venous thromboembolism (VTE) is started on enoxaparin (Lovenox) and warfarin (Coumadin). The patient asks the nurse why two medications are necessary. Which response by the nurse is most accurate? a. "Taking two blood thinners greatly reduces the risk for another clot to form." b. "Enoxaparin will work right away, but warfarin takes several days to begin preventing clots." c. "Enoxaparin will start to dissolve the clot, and warfarin will prevent any more clots from forming." d. "Because of the risk for a blood clot in the lungs, it is important for you to take more than one blood thinner."

ANS: B Low molecular weight heparin (LMWH) is used because of the immediate effect on coagulation and discontinued once the international normalized ratio (INR) value indicates that the warfarin has reached a therapeutic level. LMWH has no thrombolytic properties. The use of two anticoagulants is not related to the risk for pulmonary embolism, and two are not necessary to reduce the risk for another VTE. Anticoagulants do not thin the blood.

The nurse has started discharge teaching for a patient who is to continue warfarin (Coumadin) after hospitalization for venous thromboembolism (VTE). The nurse determines that additional teaching is needed when the patient says which of the following? a. "I should get a Medic Alert device stating that I take warfarin." b. "I should reduce the amount of green, leafy vegetables that I eat." c. "I will need routine blood tests to monitor the effects of the warfarin." d. "I will check with my health care provider before I begin any new drugs."

ANS: B Patients taking warfarin are taught to follow a consistent diet with regard to foods that are high in vitamin K, such as green, leafy vegetables. The other patient statements are accurate.

Which instructions should the nurse include in a teaching plan for an older patient newly diagnosed with peripheral artery disease (PAD)? a. "Exercise only if you do not experience any pain." b. "It is very important that you stop smoking cigarettes." c. "Try to keep your legs elevated whenever you are sitting." d. "Put elastic compression stockings on early in the morning."

ANS: B Smoking cessation is essential for slowing the progression of PAD to critical limb ischemia and reducing the risk of myocardial infarction and death. Circulation to the legs will decrease if the legs are elevated. Patients with PAD are taught to exercise to the point of feeling pain, rest, and then resume walking. Support hose are not used for patients with PAD.

After receiving change of shift report, which patient admitted to the emergency department should the nurse assess first? a. A 67-yr-old patient who has a gangrenous left foot ulcer with a weak pedal pulse b. A 50-yr-old patient who is complaining of sudden sharp and severe upper back pain c. A 39-yr-old patient who has right calf tenderness, redness, and swelling after a plane ride d. A 58-yr-old patient who is taking anticoagulants for atrial fibrillation and has black stools

ANS: B The patient's presentation of sudden sharp and severe upper back pain is consistent with dissecting thoracic aneurysm, which will require the most rapid intervention. The other patients also require rapid intervention but not before the patient with severe pain.

13. A patient inquires how something as simple as walking could help his venous vascular disorder. What is the best response by the nurse when explaining the benefits of walking? a. Improves the strength of the vascular walls b. Boosts venous circulation through leg muscle activity c. Increases cardiac output d. Clears plaques from the veins

ANS: Boosts venous circulation through leg muscle activity

16. A nurse is performing an intake examination on a patient with peripheral vascular disease (PVD). Which lifestyle information identified by the patient aggravates vascular disease? a. Riding a bicycle to work b. Drinking red wine every day c.Boosts venous circulation through leg muscle activity d. Eating chocolate candy every day

ANS: Boosts venous circulation through leg muscle activity

26. What assessment should a nurse perform on a patient after the repair of an abdominal aortic aneurysm? a. Periorbital edema b. Tremor or facial twitching c. Rising blood pressure d. Bowel sounds

ANS: Bowel sounds

4. A nurse records the assessment of stasis dermatitis on an intake assessment for a patient with peripheral vascular disease (PVD). What is the best way to describe this finding? a. Brownish skin discoloration on the lower legs b. Ulceration on medial surface of the lower legs c. Edema in the lower legs d. Purple rash on medial surface of the lower legs

ANS: Brownish skin discoloration on the lower legs

2. A 68-year-old patient is being admitted with a possible stroke. Which information from the assessment indicates that the nurse should consult with the health care provider before giving the prescribed aspirin? a. The patient has dysphasia. b. The patient has atrial fibrillation. c. The patient reports that symptoms began with a severe headache. d. The patient has a history of brief episodes of right-sided hemiplegia.

ANS: C A sudden onset headache is typical of a subarachnoid hemorrhage, and aspirin is contraindicated. Atrial fibrillation, dysphasia, and transient ischemic attack (TIA) are not contraindications to aspirin use, so the nurse can administer the aspirin. DIF: Cognitive Level: Apply (application) REF: 1392-1393 TOP: Nursing Process: Assessment MSC: NCLEX: Physiological Integrity

25. The nurse is caring for a patient who has been experiencing stroke symptoms for 60 minutes. Which action can the nurse delegate to a licensed practical/vocational nurse (LPN/LVN)? a. Assess the patient's gag and cough reflexes. b. Determine when the stroke symptoms began. c. Administer the prescribed short-acting insulin. d. Infuse the prescribed IV metoprolol (Lopressor).

ANS: C Administration of subcutaneous medications is included in LPN/LVN education and scope of practice. The other actions require more education and scope of practice and should be done by the registered nurse (RN). DIF: Cognitive Level: Apply (application) REF: 1405 OBJ: Special Questions: Delegation TOP: Nursing Process: Planning MSC: NCLEX: Safe and Effective Care Environment

17. A 72-year-old patient who has a history of a transient ischemic attack (TIA) has an order for aspirin 160 mg daily. When the nurse is administering medications, the patient says, "I don't need the aspirin today. I don't have a fever." Which action should the nurse take? a. Document that the aspirin was refused by the patient. b. Tell the patient that the aspirin is used to prevent a fever. c. Explain that the aspirin is ordered to decrease stroke risk. d. Call the health care provider to clarify the medication order.

ANS: C Aspirin is ordered to prevent stroke in patients who have experienced TIAs. Documentation of the patient's refusal to take the medication is an inadequate response by the nurse. There is no need to clarify the order with the health care provider. The aspirin is not ordered to prevent aches and pains. DIF: Cognitive Level: Apply (application) REF: 1396 TOP: Nursing Process: Implementation MSC: NCLEX: Physiological Integrity

12. The nurse identifies the nursing diagnosis of imbalanced nutrition: less than body requirements related to impaired self-feeding ability for a left-handed patient with left-sided hemiplegia. Which intervention should be included in the plan of care? a. Provide a wide variety of food choices. b. Provide oral care before and after meals. c. Assist the patient to eat with the right hand. d. Teach the patient the "chin-tuck" technique.

ANS: C Because the nursing diagnosis indicates that the patient's imbalanced nutrition is related to the left-sided hemiplegia, the appropriate interventions will focus on teaching the patient to use the right hand for self-feeding. The other interventions are appropriate for patients with other etiologies for the imbalanced nutrition. DIF: Cognitive Level: Apply (application) REF: 1407 TOP: Nursing Process: Planning MSC: NCLEX: Physiological Integrity

The nurse performing an assessment of a patient who has chronic peripheral artery disease (PAD) of the legs and an ulcer on the right second toe would expect to find a. dilated superficial veins. b. swollen dry, scaly ankles. c. prolonged capillary refill in all the toes. d. serosanguineous drainage from the ulcer.,

ANS: C Capillary refill is prolonged in PAD because of the slower and decreased blood flow to the periphery. The other listed clinical manifestations are consistent with chronic venous disease. DIF: Cognitive Level: Apply (application) REF: 807 TOP: Nursing Process: Assessment MSC: NCLEX: Physiological Integrity

11. When caring for a patient with a new right-sided homonymous hemianopsia resulting from a stroke, which intervention should the nurse include in the plan of care? a. Apply an eye patch to the right eye. b. Approach the patient from the right side. c. Place objects needed on the patient's left side. d. Teach the patient that the left visual deficit will resolve.

ANS: C During the acute period, the nurse should place objects on the patient's unaffected side. Because there is a visual defect in the right half of each eye, an eye patch is not appropriate. The patient should be approached from the left side. The visual deficit may not resolve, although the patient can learn to compensate for the defect. DIF: Cognitive Level: Apply (application) REF: 1407 TOP: Nursing Process: Planning MSC: NCLEX: Physiological Integrity

A 46-yr-old service-counter worker undergoes sclerotherapy for treatment of superficial varicose veins at an outpatient center. Which instructions should the nurse provide to the patient before discharge? a. Sitting at the work counter rather than standing, is recommended. b. Exercise, such as walking or jogging, can cause recurrence of varicosities. c. Elastic compression stockings should be applied before getting out of bed. d. Taking an aspirin daily will help prevent clots from forming around venous valves.

ANS: C Elastic compression stockings are applied with the legs elevated to reduce pressure in the lower legs. Walking is recommended to prevent recurrent varicosities. Sitting and standing are both risk factors for varicose veins and venous insufficiency. An aspirin a day is not adequate to prevent venous thrombosis and would not be recommended for a patient who had just had sclerotherapy. DIF: Cognitive Level: Apply (application) REF: 825 TOP: Nursing Process: Implementation MSC: NCLEX: Physiological Integrity

20. Which stroke risk factor for a 48-year-old male patient in the clinic is most important for the nurse to address? a. The patient is 25 pounds above the ideal weight. b. The patient drinks a glass of red wine with dinner daily. c. The patient's usual blood pressure (BP) is 170/94 mm Hg. d. The patient works at a desk and relaxes by watching television.

ANS: C Hypertension is the single most important modifiable risk factor. People who drink more than 1 (for women) or 2 (for men) alcoholic beverages a day may increase risk for hypertension. Physical inactivity and obesity contribute to stroke risk but not as much as hypertension. DIF: Cognitive Level: Apply (application) REF: 1347 OBJ: Special Questions: Prioritization TOP: Nursing Process: Assessment MSC: NCLEX: Health Promotion and Maintenance

When caring for a patient on the first postoperative day after an abdominal aortic aneurysm repair which assessment finding is most important for the nurse to communicate to the health care provider? a. Presence of flatus b. Hypoactive bowel sounds c. Maroon-colored liquid stool d. Abdominal pain with palpation

ANS: C Loose, bloody (maroon colored) stools at this time may indicate intestinal ischemia or infarction and should be reported immediately because the patient may need an emergency bowel resection. The other findings are normal on the first postoperative day after abdominal surgery. DIF: Cognitive Level: Analyze (analysis) REF: 813 OBJ: Special Questions: Prioritization TOP: Nursing Process: Assessment MSC: NCLEX: Physiological Integrity

Which assessment finding for a patient who has been admitted with a right calf venous thromboembolism (VTE) requires immediate action by the nurse? a. Erythema of right lower leg b. Complaint of right calf pain c. New onset shortness of breath d. Temperature of 100.4°F (38°C)

ANS: C New onset dyspnea suggests a pulmonary embolus, which will require rapid actions such as O2 administration and notification of the health care provider. The other findings are typical of VTE. DIF: Cognitive Level: Analyze (analysis) REF: 824 OBJ: Special Questions: Prioritization TOP: Nursing Process: Planning MSC: NCLEX: Physiological Integrity

A young adult patient tells the health care provider about experiencing cold numb fingers when running during the winter, and Raynaud's phenomenon is suspected. The nurse will anticipate teaching the patient about tests for a. hyperglycemia. b. hyperlipidemia. c. autoimmune disorders. d. coronary artery disease.

ANS: C Secondary Raynaud's phenomenon may occur in conjunction with autoimmune diseases such as rheumatoid arthritis. Patients should be screened for autoimmune disorders. Raynaud's phenomenon is not associated with hyperlipidemia, hyperglycemia, or coronary artery disease. DIF: Cognitive Level: Apply (application) REF: 809 TOP: Nursing Process: Planning MSC: NCLEX: Physiological Integrity

Which action by a new nurse who is giving fondaparinux (Arixtra) to a patient with a lower leg venous thromboembolism (VTE) indicates that more education about the drug is needed? a. The nurse avoids rubbing the injection site after giving the drug. b. The nurse injects the drug into the abdominal subcutaneous tissue. c. The nurse ejects the air bubble from the syringe before giving the drug. d. The nurse does not check partial thromboplastin time (PTT) before giving the drug.

ANS: C The air bubble is not ejected before giving fondaparinux to avoid loss of drug. The other actions by the nurse are appropriate for subcutaneous administration of a low molecular weight heparin (LMWH). LMWHs typically do not require ongoing PTT monitoring and dose adjustment. DIF: Cognitive Level: Apply (application) REF: 820 TOP: Nursing Process: Implementation MSC: NCLEX: Safe and Effective Care Environment

The nurse is caring for a patient with a descending aortic dissection. Which assessment finding is most important to report to the health care provider? a. Weak pedal pulses b. Absent bowel sounds c. Blood pressure of 138/88 mm Hg d. 25 mL of urine output over the past hour

ANS: C The blood pressure is typically kept at less than 120 mm Hg systolic to minimize extension of the dissection. The nurse will need to notify the health care provider so that -blockers or other antihypertensive drugs can be prescribed. The other findings are typical with aortic dissection and should also be reported but do not require immediate action. DIF: Cognitive Level: Analyze (analysis) REF: 815 OBJ: Special Questions: Prioritization TOP: Nursing Process: Assessment MSC: NCLEX: Physiological Integrity

15. A male patient who has right-sided weakness after a stroke is making progress in learning to use the left hand for feeding and other activities. The nurse observes that when the patient's wife is visiting, she feeds and dresses him. Which nursing diagnosis is most appropriate for the patient? a. Interrupted family processes related to effects of illness of a family member b. Situational low self-esteem related to increasing dependence on spouse for care c. Disabled family coping related to inadequate understanding by patient's spouse d. Impaired nutrition: less than body requirements related to hemiplegia and aphasia

ANS: C The information supports the diagnosis of disabled family coping because the wife does not understand the rehabilitation program. There are no data supporting low self-esteem, and the patient is attempting independence. The data do not support an interruption in family processes because this may be a typical pattern for the couple. There is no indication that the patient has impaired nutrition. DIF: Cognitive Level: Apply (application) REF: 1362 TOP: Nursing Process: Diagnosis MSC: NCLEX: Psychosocial Integrity

28. A 70-year-old female patient with left-sided hemiparesis arrives by ambulance to the emergency department. Which action should the nurse take first? a. Monitor the blood pressure. b. Send the patient for a computed tomography (CT) scan. c. Check the respiratory rate and effort. d. Assess the Glasgow Coma Scale score.

ANS: C The initial nursing action should be to assess the airway and take any needed actions to ensure a patent airway. The other activities should take place quickly after the ABCs (airway, breathing, and circulation) are completed. DIF: Cognitive Level: Apply (application) REF: 1354 OBJ: Special Questions: Prioritization TOP: Nursing Process: Implementation MSC: NCLEX: Physiological Integrity

14. A 47-year-old patient will attempt oral feedings for the first time since having a stroke. The nurse should assess the gag reflex and then a. order a varied pureed diet. b. assess the patient's appetite. c. assist the patient into a chair. d. offer the patient a sip of juice.

ANS: C The patient should be as upright as possible before attempting feeding to make swallowing easier and decrease aspiration risk. To assess swallowing ability, the nurse should initially offer water or ice to the patient. Pureed diets are not recommended because the texture is too smooth. The patient may have a poor appetite, but the oral feeding should be attempted regardless. DIF: Cognitive Level: Apply (application) REF: 1360 TOP: Nursing Process: Implementation MSC: NCLEX: Physiological Integrity

1. After a patient experienced a brief episode of tinnitus, diplopia, and dysarthria with no residual effects, the nurse anticipates teaching the patient about a. cerebral aneurysm clipping. b. heparin intravenous infusion. c. oral low-dose aspirin therapy. d. tissue plasminogen activator (tPA).

ANS: C The patient's symptoms are consistent with transient ischemic attack (TIA), and drugs that inhibit platelet aggregation are prescribed after a TIA to prevent stroke. Continuous heparin infusion is not routinely used after TIA or with acute ischemic stroke. The patient's symptoms are not consistent with a cerebral aneurysm. tPA is used only for acute ischemic stroke, not for TIA. DIF: Cognitive Level: Apply (application) REF: 1391 | 1396 TOP: Nursing Process: Planning MSC: NCLEX: Physiological Integrity

4. During the change of shift report, a nurse is told that a patient has an occluded left posterior cerebral artery. The nurse will anticipate that the patient may have a. dysphasia. b. confusion. c. visual deficits. d. poor judgment.

ANS: C Visual disturbances are expected with posterior cerebral artery occlusion. Aphasia occurs with middle cerebral artery involvement. Cognitive deficits and changes in judgment are more typical of anterior cerebral artery occlusion. *Note: However, right side causes visual deficits and left side occlusion causes dysphasia. OR is it because posterior artery leads to the occipital brain in charge of vision? DIF: Cognitive Level: Apply (application) REF: 1350 TOP: Nursing Process: Assessment MSC: NCLEX: Physiological Integrity

The nurse performing an assessment of a patient who has chronic peripheral artery disease (PAD) of the legs and an ulcer on the right second toe would expect to find a. dilated superficial veins. b. swollen, dry, scaly ankles. c. prolonged capillary refill in all the toes. d. serosanguineous drainage from the ulcer

ANS: C Capillary refill is prolonged in PAD because of the slower and decreased blood flow to the periphery. The other listed clinical manifestations are consistent with chronic venous disease

Which assessment finding for a patient who has been admitted with a right calf venous thromboembolism (VTE) requires immediate action by the nurse? a. Erythema of right lower leg c. New onset shortness of breath b. Complaint of right calf pain d. Temperature of 100.4°F (38°C)

ANS: C New onset dyspnea suggests a pulmonary embolus, which will require rapid actions such as O2 administration and notification of the health care provider. The other findings are typical of VTE

1. What is a normal age-related change in older adults that makes them susceptible to cardiovascular disease? a. Increase in cardiac output b. Increase in stroke volume c. Stiff peripheral vessels d. Oxygen capacity improvement

ANS: Stiff peripheral vessels

29. The home health nurse is caring for an 81-yr-old who had a stroke 2 months ago. Based on information shown in the accompanying figure from the history, physical assessment, and physical and occupational therapy, which problem is the highest priority? History • Well controlled type 2 diabetes for 10 years •Married 45 years; spouse has heart failure and chronic obstructive pulmonary disease Physical Assessment • Oriented to time, place, person • Speech clear •Minimal left leg weakness Physical/Occupational Therapy • Uses cane with walking • Spouse does household cleaning and cooking and assists patient with bathing and dressing a. Risk for hypoglycemia b. Impaired transfer ability c. Risk for caregiver role strain d. Ineffective health maintenance

ANS: C The spouse's household and patient care responsibilities, in combination with chronic illnesses, indicate a high risk for caregiver role strain. The nurse should further assess the situation and take appropriate actions. The data about the control of the patient's diabetes indicates that ineffective health maintenance and risk for unstable blood glucose are not priority concerns at this time. Because the patient is able to ambulate with a cane, the nursing diagnosis of impaired transfer ability is not supported. DIF: Cognitive Level: Analyze (analysis) REF: 1362 OBJ: Special Questions: Prioritization TOP: Nursing Process: Diagnosis MSC: NCLEX: Psychosocial Integrity

3. A 69-year-old patient reports a burning, aching pain in the legs when walking to the mailbox. These symptoms are relieved with rest. What should the nurse suspect? a. Venous insufficiency b. Claudication c. Phlebitis d. Rest pain

ANS: Claudication

When discussing risk factor modification for a patient who has a 5-cm abdominal aortic aneurysm the nurse will focus teaching on which patient risk factor? a. Male gender b. Turner syndrome c. Abdominal trauma history d. Uncontrolled hypertension,

ANS: D All of the factors contribute to the patient's risk but only hypertension can potentially be modified to decrease the patient's risk for further expansion of the aneurysm. DIF: Cognitive Level: Apply (application) REF: 810 TOP: Nursing Process: Implementation MSC: NCLEX: Physiological Integrity

Which nursing intervention for a patient who had an open repair of an abdominal aortic aneurysm 2 days previously is appropriate for the nurse to delegate to unlicensed assistive personnel (UAP)? a. Monitor the quality and presence of the pedal pulses. b. Teach the patient the signs of possible wound infection. c. Check the lower extremities for strength and movement. d. Help the patient to use a pillow to splint while coughing.

ANS: D Assisting a patient who has already been taught how to cough is part of routine postoperative care and within the education and scope of practice for UAP. Patient teaching and assessment of essential postoperative functions such as circulation and movement should be done by RNs. DIF: Cognitive Level: Apply (application) REF: 824 OBJ: Special Questions: Delegation TOP: Nursing Process: Planning MSC: NCLEX: Safe and Effective Care Environment

7. A patient admitted with possible stroke has been aphasic for 3 hours and his current blood pressure (BP) is 174/94 mm Hg. Which order by the health care provider should the nurse question? a. Keep head of bed elevated at least 30 degrees. b. Infuse normal saline intravenously at 75 mL/hr. c. Administer tissue plasminogen activator (tPA) per protocol. d. Administer a labetalol (Normodyne) drip to keep BP less than 140/90 mm Hg.

ANS: D Because elevated BP may be a protective response to maintain cerebral perfusion, antihypertensive therapy is recommended only if mean arterial pressure (MAP) is >130 mm Hg or systolic pressure is >220 mm Hg. Fluid intake should be 1500 to 2000 mL daily to maintain cerebral blood flow. The head of the bed should be elevated to at least 30 degrees, unless the patient has symptoms of poor tissue perfusion. tPA may be administered if the patient meets the other criteria for tPA use. DIF: Cognitive Level: Apply (application) REF: 1354 TOP: Nursing Process: Implementation MSC: NCLEX: Physiological Integrity

A patient is being evaluated for postthrombotic syndrome. Which assessment will the nurse perform? a. Ask about leg pain with exercise. b. Determine the ankle-brachial index. c. Assess capillary refill in the patient's toes. d. Inspect for presence of lipodermatosclerosis.

ANS: D Clinical signs of postthrombotic syndrome include lipodermatosclerosis. In this situation, the skin on the lower leg becomes scarred, and the leg becomes tapered like an "inverted bottle." The other assessments would be done for patients with peripheral arterial disease. DIF: Cognitive Level: Apply (application) REF: 818 TOP: Nursing Process: Evaluation MSC: NCLEX: Physiological Integrity

Which topic should the nurse include in patient teaching for a patient with a venous stasis ulcer on the left lower leg? a. Need to increase carbohydrate intake b. Methods of keeping the wound area dry c. Purpose of prophylactic antibiotic therapy d. Application of elastic compression stockings

ANS: D Compression of the leg is essential to healing of venous stasis ulcers. High dietary intake of protein, rather than carbohydrates, is needed. Prophylactic antibiotics are not routinely used for venous ulcers. Moist dressings are used to hasten wound healing. DIF: Cognitive Level: Apply (application) REF: 826 TOP: Nursing Process: Planning MSC: NCLEX: Physiological Integrity

19. A 58-year-old patient with a left-brain stroke suddenly bursts into tears when family members visit. The nurse should a. use a calm voice to ask the patient to stop the crying behavior. b. explain to the family that depression is normal following a stroke. c. have the family members leave the patient alone for a few minutes. d. teach the family that emotional outbursts are common after strokes.

ANS: D Patients who have left-sided brain stroke are prone to emotional outbursts that are not necessarily related to the emotional state of the patient. Depression after a stroke is common, but the suddenness of the patient's outburst suggests that depression is not the major cause of the behavior. The family should stay with the patient. The crying is not within the patient's control and asking the patient to stop will lead to embarrassment. DIF: Cognitive Level: Apply (application) REF: 1364 TOP: Nursing Process: Implementation MSC: NCLEX: Psychosocial Integrity

2. What should a nurse ask a patient related to past history of deep-vein thrombosis (DVT) and other vascular problems? a. An aneurysm b. Rheumatoid arthritis c. A peptic ulcer d. Stiff peripheral vessels

ANS: Stiff peripheral vessels

8. A nurse records that a patient has a 3+ edema to the right foot. How deep did the nurse's thumb depress the edematous area? a. More than 1 inch b. To 1 inch c. To inch d. Less than inch

ANS: To 1 inch

3. The etiology of an ischemic stroke would include a(an): a. cardiogenic emboli. b. cerebral aneurysm. c. arteriovenous malformation. d. intracerebral hemorrhage.

a

23. Nurses in change-of-shift report are discussing the care of a patient with a stroke who has progressively increasing weakness and decreasing level of consciousness (LOC). Which nursing diagnosis do they determine has thehighest priority for the patient? a. Impaired physical mobility related to weakness b. Disturbed sensory perception related to brain injury c. Risk for impaired skin integrity related to immobility d. Risk for aspiration related to inability to protect airway

ANS: D Protection of the airway is the priority of nursing care for a patient having an acute stroke. The other diagnoses are also appropriate, but interventions to prevent aspiration are the priority at this time. DIF: Cognitive Level: Apply (application) REF: 1397 | 1401 OBJ: Special Questions: Prioritization TOP: Nursing Process: Analysis MSC: NCLEX: Physiological Integrity

22. A patient with left-sided weakness that started 60 minutes earlier is admitted to the emergency department and diagnostic tests are ordered. Which test should be done first? a. Complete blood count (CBC) b. Chest radiograph (Chest x-ray) c. 12-Lead electrocardiogram (ECG) d. Non contrast computed tomography (CT) scan

ANS: D Rapid screening with a noncontrast CT scan is needed before administration of tissue plasminogen activator (tPA), which must be given within 4.5 hours of the onset of clinical manifestations of the stroke. The sooner the tPA is given, the less brain injury. The other diagnostic tests give information about possible causes of the stroke and do not need to be completed as urgently as the CT scan. DIF: Cognitive Level: Apply (application) REF: 1354 OBJ: Special Questions: Prioritization TOP: Nursing Process: Implementation MSC: NCLEX: Physiological Integrity

3. A 73-year-old patient with a stroke experiences facial drooping on the right side and right-sided arm and leg paralysis. When admitting the patient, which clinical manifestation will the nurse expect to find? a. Impulsive behavior b. Right-sided neglect c. Hyperactive left-sided tendon reflexes d. Difficulty comprehending instructions

ANS: D Right-sided paralysis indicates a left-brain stroke, which will lead to difficulty with comprehension and use of language. The left-side reflexes are likely to be intact. Impulsive behavior and neglect are more likely with a right-side stroke. DIF: Cognitive Level: Apply (application) REF: 1407 TOP: Nursing Process: Assessment MSC: NCLEX: Physiological Integrity

Several hours after a patient had an open surgical repair of an abdominal aortic aneurysm the UAP reports to the nurse that urinary output for the past 2 hours has been 45 mL. The nurse notifies the health care provider and anticipates an order for a(n) a. hemoglobin count. b. additional antibiotic. c. serum creatinine level. d. increased IV infusion rate.

ANS: D The decreased urine output suggests decreased renal perfusion and monitoring of renal function is needed. There is no indication that infection is a concern, so antibiotic therapy and a WBC count are not needed. The IV rate may be increased because hypovolemia may be contributing to the patient's decreased urinary output. DIF: Cognitive Level: Apply (application) REF: 811 TOP: Nursing Process: Planning MSC: NCLEX: Physiological Integrity

A patient at the clinic says "I always walk after dinner but lately my leg cramps and hurts after just a few minutes of starting. The pain goes away after I stop walking though." The nurse should a. look for the presence of tortuous veins bilaterally on the legs. b. ask about any skin color changes that occur in response to cold. c. assess for unilateral swelling, redness, and tenderness of either leg. d. palpate for the presence of dorsalis pedis and posterior tibial pulses.

ANS: D The nurse should assess for other clinical manifestations of peripheral arterial disease in a patient who describes intermittent claudication. Changes in skin color that occur in response to cold are consistent with Raynaud's phenomenon. Tortuous veins on the legs suggest venous insufficiency. Unilateral leg swelling, redness, and tenderness indicate venous thromboembolism. DIF: Cognitive Level: Apply (application) REF: 814 TOP: Nursing Process: Assessment MSC: NCLEX: Physiological Integrity

An older patient with chronic atrial fibrillation develops sudden severe pain pulselessness, pallor, and coolness in the right leg. The nurse should notify the health care provider and immediately a. apply a compression stocking to the leg. b. elevate the leg above the level of the heart. c. assist the patient in gently exercising the leg. d. keep the patient in bed in the supine position.

ANS: D The patient's history and clinical manifestations are consistent with acute arterial occlusion, and resting the leg will decrease the O2 demand of the tissues and minimize ischemic damage until circulation can be restored. Elevating the leg or applying an elastic wrap will further compromise blood flow to the leg. Exercise will increase oxygen demand for the tissues of the leg. DIF: Cognitive Level: Apply (application) REF: 808 TOP: Nursing Process: Implementation MSC: NCLEX: Physiological Integrity

8. A patient arrives in the emergency department with hemiparesis and dysarthria that started 2 hours previously, and health records show a history of several transient ischemic attacks (TIAs). The nurse anticipates preparing the patient for a. surgical endarterectomy. b. transluminal angioplasty. c. intravenous heparin administration. d. tissue plasminogen activator (tPA) infusion.

ANS: D The patient's history and clinical manifestations suggest an acute ischemic stroke and a patient who is seen within 4.5 hours of stroke onset is likely to receive tPA (after screening with a CT scan). Heparin administration in the emergency phase is not indicated. Emergent carotid transluminal angioplasty or endarterectomy is not indicated for the patient who is having an acute ischemic stroke. DIF: Cognitive Level: Apply (application) REF: 1391 | 1398 TOP: Nursing Process: Planning MSC: NCLEX: Physiological Integrity

The health care provider has prescribed bed rest with the feet elevated for a patient admitted to the hospital with venous thromboembolism. Which action by the nurse to elevate the patient's feet is best? a. The patient is placed in the Trendelenburg position. b. Two pillows are positioned under the affected leg. c. The bed is elevated at the knee and pillows are placed under the feet. d. One pillow is placed under the thighs and two pillows are placed under the lower legs.

ANS: D The purpose of elevating the feet is to enhance venous flow from the feet to the right atrium, which is best accomplished by placing two pillows under the feet and one under the thighs. Placing the patient in the Trendelenburg position will lower the head below heart level, which is not indicated for this patient. Placing pillows under the calf or elevating the bed at the knee may cause blood stasis at the calf level. DIF: Cognitive Level: Analyze (analysis) REF: 819 TOP: Nursing Process: Implementation MSC: NCLEX: Physiological Integrity

5. The nurse knows that symptoms associated with a transient ischemic attack (TIA), usually a precursor of a future stroke, usually subside in: a. 1 hour. b. 3 to 6 hours. c. 12 hours. d. 24 to 36 hours.

a

21. A patient in the emergency department with sudden-onset right-sided weakness is diagnosed with an intracerebral hemorrhage. Which information about the patient is most important to communicate to the health care provider? a. The patient's speech is difficult to understand. b. The patient's blood pressure is 144/90 mm Hg. c. The patient takes a diuretic because of a history of hypertension. d. The patient has atrial fibrillation and takes warfarin (Coumadin).

ANS: D The use of warfarin probably contributed to the intracerebral bleeding and remains a risk factor for further bleeding. Administration of vitamin K is needed to reverse the effects of the warfarin, especially if the patient is to have surgery to correct the bleeding. The history of hypertension is a risk factor for the patient but has no immediate effect on the patient's care. The BP of 144/90 indicates the need for ongoing monitoring but not for any immediate change in therapy. Slurred speech is consistent with a left-sided stroke, and no change in therapy is indicated. DIF: Cognitive Level: Apply (application) REF: 1349 OBJ: Special Questions: Prioritization TOP: Nursing Process: Assessment MSC: NCLEX: Physiological Integrity

Which topic should the nurse include in patient teaching for a patient with a venous stasis ulcer on the left lower leg? a. Need to increase carbohydrate intake b. Methods of keeping the wound area dry c. Purpose of prophylactic antibiotic therapy d. Application of elastic compression stockings

ANS: D Compression of the leg is essential to healing of venous stasis ulcers. High dietary intake of protein, rather than carbohydrates, is needed. Prophylactic antibiotics are not routinely used for venous ulcers. Moist dressings are used to hasten wound healing

A patient at the clinic says, "I always walk after dinner, but lately my leg cramps and hurts after just a few minutes of starting. The pain goes away after I stop walking, though." The nurse should a. look for the presence of tortuous veins bilaterally on the legs. b. ask about any skin color changes that occur in response to cold. c. assess for unilateral swelling, redness, and tenderness of either leg. d. palpate for the presence of dorsalis pedis and posterior tibial pulses.

ANS: D The nurse should assess for other clinical manifestations of peripheral arterial disease in a patient who describes intermittent claudication. Changes in skin color that occur in response to cold are consistent with Raynaud's phenomenon. Tortuous veins on the legs suggest venous insufficiency. Unilateral leg swelling, redness, and tenderness indicate venous thromboembolism.

An older patient with chronic atrial fibrillation develops sudden severe pain, pulselessness, pallor, and coolness in the right leg. The nurse should notify the health care provider and immediately a. apply a compression stocking to the leg. b. elevate the leg above the level of the heart. c. assist the patient in gently exercising the leg. d. keep the patient in bed in the supine position.

ANS: D The patient's history and clinical manifestations are consistent with acute arterial occlusion, and resting the leg will decrease the O2 demand of the tissues and minimize ischemic damage until circulation can be restored. Elevating the leg or applying an elastic wrap will further compromise blood flow to the leg. Exercise will increase oxygen demand for the tissues of the

The health care provider has prescribed bed rest with the feet elevated for a patient admitted to the hospital with venous thromboembolism. Which action by the nurse to elevate the patient's feet is best? a. The patient is placed in the Trendelenburg position. b. Two pillows are positioned under the affected leg. c. The bed is elevated at the knee and pillows are placed under the feet. d. One pillow is placed under the thighs and two pillows are placed under the lower legs.

ANS: D The purpose of elevating the feet is to enhance venous flow from the feet to the right atrium, which is best accomplished by placing two pillows under the feet and one under the thighs. Placing the patient in the Trendelenburg position will lower the head below heart level, which is not indicated for this patient. Placing pillows under the calf or elevating the bed at the knee may cause blood stasis at the calf level.

24. Which postoperative sign should a nurse report immediately when caring for a patient with an endarterectomy with a synthetic graft? a. Headache b. Fever c. Edema d. Pain

ANS: Fever

18. Which instruction is most appropriate for a patient with arterial insufficiency? a. Frequently allow the legs to dangle dependently. b. Rub the legs vigorously. c. Stand often to keep blood flow in the legs. d. Walk barefoot.

ANS: Frequently allow the legs to dangle dependently.

12. A nurse is educating a patient regarding a stress test on a treadmill. Teaching includes that this test is a noninvasive procedure. What additional information is appropriate for the nurse to include? a. Is monitored continuously by blood pressure and an electrocardiogram b. Will last about 1 hour c. Is meant to stimulate claudication and dyspnea d. Will require a period of bedrest afterward

ANS: Is monitored continuously by blood pressure and an electrocardiogram

19. A nurse is preparing to administer low-molecular-weight heparin (LMWH). What is a major advantage related to the administration of LMWH? a. It can be given orally. b. It is provided fixed doses. c. It is given only after partial thromboplastin time (PTT) laboratory work. d. It provides an immediate effect.

ANS: It is provided fixed doses

14. What is contraindicated for a patient performing Buerger-Allen exercises? a. Lying on the stomach b. Raising legs for 2 minutes until they blanch c. Lowering the legs until the color returns d. Keeping legs flat for 5 minutes and then repeat the exercise

ANS: Lying on the stomach

6. A nurse performs Homans maneuver by flexing the knee and sharply dorsiflexing the foot. What response indicates a positive Homans sign? a. Cramping of the toes b. Resisting dorsiflexion c. Pain in the calf area d. Blanching of the sole

ANS: Pain in the calf area

11. A nurse is caring for a patient who has had an angiogram. What should the nurse make a point of care to assess and document on this patient? a. Fluid intake b. Peripheral pulses in the affected leg c. Inquiring about an allergy to iodine d. Decreased blood pressure

ANS: Peripheral pulses in the affected leg

17. Vascular disease disorders often require the use of elastic stockings. Which action should the nurse implement when caring for a patient with elastic stockings? a. Apply the stockings and roll down the cuff. b. Remove the stockings for skin inspection two times a day. c. Remove the stockings when the patient is ambulating. d. Inspect the skin for pressure or irritation daily.

ANS: Remove the stockings for skin inspection two times a day.

A patient has left-sided hemiplegia after an ischemic stroke 4 days earlier. How should the nurse promote skin integrity? Position the patient on her weak side most of the time. Avoid the use of pillows to promote independence in positioning. Alternate the patient's positioning between supine and side-lying. Establish a schedule for the massage of areas where skin breakdown emerges.

Alternate the patient's positioning between supine and side-lying. A position change schedule should be established for stroke patients. An example is side-back-side, with a maximum duration of 2 hours for any position. The patient should be positioned on the weak or paralyzed side for only 30 minutes. Pillows may be used to facilitate positioning. Areas of skin breakdown should never be massaged.

The clinic nurse is caring for a 57-year-old client who reports experiencing leg pain whenever she walks several blocks. The patient has type 1 diabetes and has smoked a pack of cigarettes every day for the past 40 years. The physician diagnoses intermittent claudication. The nurse should provide what instruction about long-term care to the client? A) "Be sure to practice meticulous foot care." B) "Consider cutting down on your smoking." C) "Reduce your activity level to accommodate your limitations." D) "Try to make sure you eat enough protein."

Ans: "Be sure to practice meticulous foot care." Feedback: The patient with peripheral vascular disease or diabetes should receive education or reinforcement about skin and foot care. Intermittent claudication and other chronic peripheral vascular diseases reduce oxygenation to the feet, making them susceptible to injury and poor healing; therefore, meticulous foot care is essential. The patient should stop smoking—not just cut down—because nicotine is a vasoconstrictor. Daily walking benefits the patient with intermittent claudication. Increased protein intake will not alleviate the patient's symptoms.

A nurse in the rehabilitation unit is caring for an older adult patient who is in cardiac rehabilitation following an MI. The nurse's plan of care calls for the patient to walk for 10 minutes 3 times a day. The patient questions the relationship between walking and heart function. How should the nurse best reply? A) "The arteries in your legs constrict when you walk and allow the blood to move faster and with more pressure on the tissue." B) Walking increases your heart rate and blood pressure. Therefore your heart is under less stress." C) "Walking helps your heart adjust to your new arteries and helps build your self-esteem." D) "When you walk, the muscles in your legs contract and pump the blood in your veins back toward your heart, which allows more blood to return to your heart."

Ans: "When you walk, the muscles in your legs contract and pump the blood in your veins back toward your heart, which allows more blood to return to your heart." Feedback: Veins, unlike arteries, are equipped with valves that allow blood to move against the force of gravity. The legs have one-way bicuspid valves that prevent blood from seeping backward as it moves forward by the muscles in our legs pressing on the veins as we walk and increasing venous return. Leg arteries do constrict when walking, which allows the blood to move faster and with more pressure on the tissue, but the greater concern is increasing the flow of venous blood to the heart. Walking increases, not decreases, the heart' pumping ability, which increases heart rate and blood pressure and the hearts ability to manage stress. Walking does help the heart adjust to new arteries and may enhance self-esteem, but the patient had an MI—there are no "new arteries."

An older adult patient has been treated for a venous ulcer and a plan is in place to prevent the occurrence of future ulcers. What should the nurse include in this plan? A) Use of supplementary oxygen to aid tissue oxygenation B) Daily use of normal saline compresses on the lower limbs C) Daily administration of prophylactic antibiotics D) A high-protein diet that is rich in vitamins

Ans: A high-protein diet that is rich in vitamins Feedback: A diet that is high in protein, vitamins C and A, iron, and zinc is encouraged to promote healing and prevent future ulcers. Prophylactic antibiotics and saline compresses are not used to prevent ulcers. Oxygen supplementation does not prevent ulcer formation.

A medical nurse has admitted four patients over the course of a 12-hour shift. For which patient would assessment of ankle-brachial index (ABI) be most clearly warranted? A) A patient who has peripheral edema secondary to chronic heart failure B) An older adult patient who has a diagnosis of unstable angina C) A patient with poorly controlled type 1 diabetes who is a smoker D) A patient who has community-acquired pneumonia and a history of COPD

Ans: A patient with poorly controlled type 1 diabetes who is a smoker Feedback: Nurses should perform a baseline ABI on any patient with decreased pulses or any patient 50 years of age or older with a history of diabetes or smoking. The other answers do not apply.

The nurse is caring for a 72-year-old patient who is in cardiac rehabilitation following heart surgery. The patient has been walking on a regular basis for about a week and walks for 15 minutes 3 times a day. The patient states that he is having a cramp-like pain in the legs every time he walks and that the pain gets "better when I rest." The patient's care plan should address what problem? A) Decreased mobility related to VTE B) Acute pain related to intermittent claudication C) Decreased mobility related to venous insufficiency D) Acute pain related to vasculitis

Ans: Acute pain related to intermittent claudication Feedback: Intermittent claudication presents as a muscular, cramp-type pain in the extremities consistently reproduced with the same degree of exercise or activity and relieved by rest. Patients with peripheral arterial insufficiency often complain of intermittent claudication due to a lack of oxygen to muscle tissue. Venous insufficiency presents as a disorder of venous blood reflux and does not present with cramp-type pain with exercise. Vasculitis is an inflammation of the blood vessels and presents with weakness, fever, and fatigue, but does not present with cramp-type pain with exercise. The pain associated with VTE does not have this clinical presentation.

A nurse on a medical unit is caring for a patient who has been diagnosed with lymphangitis. When reviewing this patient's medication administration record, the nurse should anticipate which of the following? A) Coumadin (warfarin) B) Lasix (furosemide) C) An antibiotic D) An antiplatelet aggregator

Ans: An antibiotic Feedback: Lymphangitis is an acute inflammation of the lymphatic channels caused by an infectious process. Antibiotics are always a component of treatment. Diuretics are of nominal use. Anticoagulants and antiplatelet aggregators are not indicated in this form of infection.

The triage nurse in the ED is assessing a patient who has presented with complaint of pain and swelling in her right lower leg. The patient's pain became much worse last night and appeared along with fever, chills, and sweating. The patient states, "I hit my leg on the car door 4 or 5 days ago and it has been sore ever since." The patient has a history of chronic venous insufficiency. What intervention should the nurse anticipate for this patient? A) Platelet transfusion to treat thrombocytopenia B) Warfarin to treat arterial insufficiency C) Antibiotics to treat cellulitis D) Heparin IV to treat VTE

Ans: Antibiotics to treat cellulitis Feedback: Cellulitis is the most common infectious cause of limb swelling. The signs and symptoms include acute onset of swelling, localized redness, and pain; it is frequently associated with systemic signs of fever, chills, and sweating. The patient may be able to identify a trauma that accounts for the source of infection. Thrombocytopenia is a loss or decrease in platelets and increases a patient's risk of bleeding; this problem would not cause these symptoms. Arterial insufficiency would present with ongoing pain related to activity. This patient does not have signs and symptoms of VTE.

A 79-year-old man is admitted to the medical unit with digital gangrene. The man states that his problems first began when he stubbed his toe going to the bathroom in the dark. In addition to this trauma, the nurse should suspect that the patient has a history of what health problem? A) Raynaud's phenomenon B) CAD C) Arterial insufficiency D) Varicose veins

Ans: Arterial insufficiency Feedback: Arterial insufficiency may result in gangrene of the toe (digital gangrene), which usually is caused by trauma. The toe is stubbed and then turns black. Raynaud's, CAD and varicose veins are not the usual causes of digital gangrene in the elderly.

A patient who has undergone a femoral to popliteal bypass graft surgery returns to the surgical unit. Which assessments should the nurse perform during the first postoperative day? A) Assess pulse of affected extremity every 15 minutes at first. B) Palpate the affected leg for pain during every assessment. C) Assess the patient for signs and symptoms of compartment syndrome every 2 hours. D) Perform Doppler evaluation once daily.

Ans: Assess pulse of affected extremity every 15 minutes at first. Feedback: The primary objective in the postoperative period is to maintain adequate circulation through the arterial repair. Pulses, Doppler assessment, color and temperature, capillary refill, and sensory and motor function of the affected extremity are checked and compared with those of the other extremity; these values are recorded initially every 15 minutes and then at progressively longer intervals if the patient's status remains stable. Doppler evaluations should be performed every 2 hours. Pain is regularly assessed, but palpation is not the preferred method of performing this assessment. Compartment syndrome results from the placement of a cast, not from vascular surgery.

A nurse is creating an education plan for a patient with venous insufficiency. What measure should the nurse include in the plan? A) Avoiding tight-fitting socks. B) Limit activity whenever possible. C) Sleep with legs in a dependent position. D) Avoid the use of pressure stockings.

Ans: Avoiding tight-fitting socks. Feedback: Measures taken to prevent complications include avoiding tight-fitting socks and panty girdles; maintaining activities, such as walking, sleeping with legs elevated, and using pressure stockings. Not included in the teaching plan for venous insufficiency would be reducing activity, sleeping with legs dependent, and avoiding pressure stockings. Each of these actions exacerbates venous insufficiency.

A nurse is admitting a 45-year-old man to the medical unit who has a history of PAD. While providing his health history, the patient reveals that he smokes about two packs of cigarettes a day, has a history of alcohol abuse, and does not exercise. What would be the priority health education for this patient? A) The lack of exercise, which is the main cause of PAD. B) The likelihood that heavy alcohol intake is a significant risk factor for PAD. C) Cigarettes contain nicotine, which is a powerful vasoconstrictor and may cause or aggravate PAD. D) Alcohol suppresses the immune system, creates high glucose levels, and may cause PAD.

Ans: Cigarettes contain nicotine, which is a powerful vasoconstrictor and may cause or aggravate PAD. Feedback: Tobacco is powerful vasoconstrictor; its use with PAD is highly detrimental, and patients are strongly advised to stop using tobacco. Sedentary lifestyle is also a risk factor, but smoking is likely a more significant risk factor that the nurse should address. Alcohol use is less likely to cause PAD, although it carries numerous health risks.

A nurse in a long-term care facility is caring for an 83-year-old woman who has a history of HF and peripheral arterial disease (PAD). At present the patient is unable to stand or ambulate. The nurse should implement measures to prevent what complication? A) Aoritis B) Deep vein thrombosis C) Thoracic aortic aneurysm D) Raynaud's disease

Ans: Deep vein thrombosis Feedback: Although the exact cause of venous thrombosis remains unclear, three factors, known as Virchow's triad, are believed to play a significant role in its development: stasis of blood (venous stasis), vessel wall injury, and altered blood coagulation. In this woman's case, she has venous stasis from immobility, vessel wall injury from PAD, and altered blood coagulation from HF. The cause of aoritis is unknown, but it has no direct connection to HF, PAD, or mobility issues. The greatest risk factors for thoracic aortic aneurysm are atherosclerosis and hypertension; there is no direct connection to HF, PAD, or mobility issues. Raynaud's disease is a disorder that involves spasms of blood vessels and, again, no direct connection to HF, PAD, or mobility issues.

When assessing venous disease in a patient's lower extremities, the nurse knows that what test will most likely be ordered? A) Duplex ultrasonography B) Echocardiography C) Positron emission tomography (PET) D) Radiography

Ans: Duplex ultrasonography Feedback: Duplex ultrasound may be used to determine the level and extent of venous disease as well as its chronicity. Radiographs (x-rays), PET scanning, and echocardiography are never used for this purpose as they do not allow visualization of blood flow.

A postsurgical patient has illuminated her call light to inform the nurse of a sudden onset of lower leg pain. On inspection, the nurse observes that the patient's left leg is visibly swollen and reddened. What is the nurse's most appropriate action? A) Administer a PRN dose of subcutaneous heparin. B) Inform the physician that the patient has signs and symptoms of VTE. C) Mobilize the patient promptly to dislodge any thrombi in the patient's lower leg. D) Massage the patient's lower leg to temporarily restore venous return.

Ans: Inform the physician that the patient has signs and symptoms of VTE. Feedback: VTE requires prompt medical follow-up. Heparin will not dissolve an established clot. Massaging the patient's leg and mobilizing the patient would be contraindicated because they would dislodge the clot, possibly resulting in a pulmonary embolism.

The nurse is admitting a 32-year-old woman to the presurgical unit. The nurse learns during the admission assessment that the patient takes oral contraceptives. Consequently, the nurse's postoperative plan of care should include what intervention? A) Early ambulation and leg exercises B) Cessation of the oral contraceptives until 3 weeks postoperative C) Doppler ultrasound of peripheral circulation twice daily D) Dependent positioning of the patient's extremities when at rest

Ans: Early ambulation and leg exercises Feedback: Oral contraceptive use increases blood coagulability; with bed rest, the patient may be at increased risk of developing deep vein thrombosis. Leg exercises and early ambulation are among the interventions that address this risk. Assessment of peripheral circulation is important, but Doppler ultrasound may not be necessary to obtain these data. Dependent positioning increases the risk of venous thromboembolism (VTE). Contraceptives are not normally discontinued to address the risk of VTE in the short term.

The nurse is evaluating a patient's diagnosis of arterial insufficiency with reference to the adequacy of the patient's blood flow. On what physiological variables does adequate blood flow depend? Select all that apply. A) Efficiency of heart as a pump B) Adequacy of circulating blood volume C) Ratio of platelets to red blood cells D) Size of red blood cells E) Patency and responsiveness of the blood vessels

Ans: Efficiency of heart as a pump, Adequacy of circulating blood volume, Patency and responsiveness of the blood vessels Feedback: Adequate blood flow depends on the efficiency of the heart as a pump, the patency and responsiveness of the blood vessels, and the adequacy of circulating blood volume. Adequacy of blood flow does not primarily depend on the size of red cells or their ratio to the number of platelets.

How should the nurse best position a patient who has leg ulcers that are venous in origin? A) Keep the patient's legs flat and straight. B) Keep the patient's knees bent to 45-degree angle and supported with pillows. C) Elevate the patient's lower extremities. D) Dangle the patient's legs over the side of the bed.

Ans: Elevate the patient's lower extremities. Feedback: Positioning of the legs depends on whether the ulcer is of arterial or venous origin. With venous insufficiency, dependent edema can be avoided by elevating the lower extremities. Dangling the patient's legs and applying pillows may further compromise venous return.

A nurse has written a plan of care for a man diagnosed with peripheral arterial insufficiency. One of the nursing diagnoses in the care plan is altered peripheral tissue perfusion related to compromised circulation. What is the most appropriate intervention for this diagnosis? A) Elevate his legs and arms above his heart when resting. B) Encourage the patient to engage in a moderate amount of exercise. C) Encourage extended periods of sitting or standing. D) Discourage walking in order to limit pain.

Ans: Encourage the patient to engage in a moderate amount of exercise. Feedback: The nursing diagnosis of altered peripheral tissue perfusion related to compromised circulation requires interventions that focus on improving circulation. Encouraging the patient to engage in a moderate amount of exercise serves to improve circulation. Elevating his legs and arms above his heart when resting would be passive and fails to promote circulation. Encouraging long periods of sitting or standing would further compromise circulation. The nurse should encourage, not discourage, walking to increase circulation and decrease pain.

A patient with advanced venous insufficiency is confined following orthopedic surgery. How can the nurse best prevent skin breakdown in the patient's lower extremities? A) Ensure that the patient's heels are protected and supported. B) Closely monitor the patient's serum albumin and prealbumin levels. C) Perform gentle massage of the patient's lower legs, as tolerated. D) Perform passive range-of-motion exercises once per shift.

Ans: Ensure that the patient's heels are protected and supported. Feedback: If the patient is on bed rest, it is important to relieve pressure on the heels to prevent pressure ulcerations, since the heels are among the most vulnerable body regions. Monitoring blood work does not directly prevent skin breakdown, even though albumin is related to wound healing. Massage is not normally indicated and may exacerbate skin breakdown. Passive range- of-motion exercises do not directly reduce the risk of skin breakdown.

The nurse is caring for a patient who is admitted to the medical unit for the treatment of a venous ulcer in the area of her lateral malleolus that has been unresponsive to treatment. What is the nurse most likely to find during an assessment of this patient's wound? A) Hemorrhage B) Heavy exudate C) Deep wound bed D) Pale-colored wound bed

Ans: Heavy exudate Feedback: Venous ulcerations in the area of the medial or lateral malleolus (gaiter area) are typically large, superficial, and highly exudative. Venous hypertension causes extravasation of blood, which discolors the area of the wound bed. Bleeding is not normally present.

The nurse caring for a patient with a leg ulcer has finished assessing the patient and is developing a problem list prior to writing a plan of care. What major nursing diagnosis might the care plan include? A) Risk for disuse syndrome B) Ineffective health maintenance C) Sedentary lifestyle D) Imbalanced nutrition: less than body requirements

Ans: Imbalanced nutrition: less than body requirements Feedback: Major nursing diagnoses for the patient with leg ulcers may include imbalanced nutrition: less than body requirements, related to increased need for nutrients that promote wound healing. Risk for disuse syndrome is a state in which an individual is at risk for deterioration of body systems owing to prescribed or unavoidable musculoskeletal inactivity. A leg ulcer will affect activity, but rarely to this degree. Leg ulcers are not necessarily a consequence of ineffective health maintenance or sedentary lifestyle.

While assessing a patient the nurse notes that the patient's ankle-brachial index (ABI) of the right leg is 0.40. How should the nurse best respond to this assessment finding? A) Assess the patient's use of over-the-counter dietary supplements. B) Implement interventions relevant to arterial narrowing. C) Encourage the patient to increase intake of foods high in vitamin K. D) Adjust the patient's activity level to accommodate decreased coronary output.

Ans: Implement interventions relevant to arterial narrowing. Feedback: ABI is used to assess the degree of stenosis of peripheral arteries. An ABI of less than 1.0 indicates possible claudication of the peripheral arteries. It does not indicate inadequate coronary output. There is no direct indication for changes in vitamin K intake and OTC medications are not likely causative.

The nurse is taking a health history of a new patient. The patient reports experiencing pain in his left lower leg and foot when walking. This pain is relieved with rest. The nurse notes that the left lower leg is slightly edematous and is hairless. When planning this patient's subsequent care, the nurse should most likely address what health problem? A) Coronary artery disease (CAD) B) Intermittent claudication C) Arterial embolus D) Raynaud's disease

Ans: Intermittent claudication Feedback: A muscular, cramp-type pain in the extremities consistently reproduced with the same degree of exercise or activity and relieved by rest is experienced by patients with peripheral arterial insufficiency. Referred to as intermittent claudication, this pain is caused by the inability of the arterial system to provide adequate blood flow to the tissues in the face of increased demands for nutrients and oxygen during exercise. The nurse would not suspect the patient has CAD, arterial embolus, or Raynaud's disease; none of these health problems produce this cluster of signs and symptoms.

The nurse is preparing to administer warfarin (Coumadin) to a client with deep vein thrombophlebitis (DVT). Which laboratory value would most clearly indicate that the patient's warfarin is at therapeutic levels? A) Partial thromboplastin time (PTT) within normal reference range B) Prothrombin time (PT) eight to ten times the control C) International normalized ratio (INR) between 2 and 3 D) Hematocrit of 32%

Ans: International normalized ratio (INR) between 2 and 3 Feedback: The INR is most often used to determine if warfarin is at a therapeutic level; an INR of 2 to 3 is considered therapeutic. Warfarin is also considered to be at therapeutic levels when the client's PT is 1.5 to 2 times the control. Higher values indicate increased risk of bleeding and hemorrhage, whereas lower values indicate increased risk of blood clot formation. Heparin, not warfarin, prolongs PTT. Hematocrit does not provide information on the effectiveness of warfarin; however, a falling hematocrit in a client taking warfarin may be a sign of hemorrhage.

The nurse has performed a thorough nursing assessment of the care of a patient with chronic leg ulcers. The nurse's assessment should include which of the following components? Select all that apply. A) Location and type of pain B) Apical heart rate C) Bilateral comparison of peripheral pulses D) Comparison of temperature in the patient's legs E) Identification of mobility limitations

Ans: Location and type of pain, Bilateral comparison of peripheral pulses, Comparison of temperature in the patient's legs, Identification of mobility limitations Feedback: A careful nursing history and assessment are important. The extent and type of pain are carefully assessed, as are the appearance and temperature of the skin of both legs. The quality of all peripheral pulses is assessed, and the pulses in both legs are compared. Any limitation of mobility and activity that results from vascular insufficiency is identified. Not likely is there any direct indication for assessment of apical heart rate, although peripheral pulses must be assessed.

The nurse is caring for an acutely ill patient who is on anticoagulant therapy. The patient has a comorbidity of renal insufficiency. How will this patient's renal status affect heparin therapy? A) Heparin is contraindicated in the treatment of this patient. B) Heparin may be administered subcutaneously, but not IV. C) Lower doses of heparin are required for this patient. D) Coumadin will be substituted for heparin.

Ans: Lower doses of heparin are required for this patient. Feedback: If renal insufficiency exists, lower doses of heparin are required. Coumadin cannot be safely and effectively used as a substitute and there is no contraindication for IV administration.

A patient comes to the walk-in clinic with complaints of pain in his foot following stepping on a roofing nail 4 days ago. The patient has a visible red streak running up his foot and ankle. What health problem should the nurse suspect? A) Cellulitis B) Local inflammation C) Elephantiasis D) Lymphangitis

Ans: Lymphangitis Feedback: Lymphangitis is an acute inflammation of the lymphatic channels. It arises most commonly from a focus of infection in an extremity. Usually, the infectious organism is hemolytic streptococcus. The characteristic red streaks that extend up the arm or the leg from an infected wound outline the course of the lymphatic vessels as they drain. Cellulitis is caused by bacteria, which cause a generalized edema in the subcutaneous tissues surrounding the affected area. Local inflammation would not present with red streaks in the lymphatic channels. Elephantiasis is transmitted by mosquitoes that carry parasitic worm larvae; the parasites obstruct the lymphatic channels and results in gross enlargement of the limbs.

A nurse working in a long-term care facility is performing the admission assessment of a newly admitted, 85-year-old resident. During inspection of the resident's feet, the nurse notes that she appears to have early evidence of gangrene on one of her great toes. The nurse knows that gangrene in the elderly is often the first sign of what? A) Chronic venous insufficiency B) Raynaud's phenomenon C) VTE D) PAD

Ans: PAD Feedback: In elderly people, symptoms of PAD may be more pronounced than in younger people. In elderly patients who are inactive, gangrene may be the first sign of disease. Venous insufficiency does not normally manifest with gangrene. Similarly, VTE and Raynaud's phenomenon do not cause the ischemia that underlies gangrene.

The nurse is caring for a patient with a large venous leg ulcer. What intervention should the nurse implement to promote healing and prevent infection? A) Provide a high-calorie, high-protein diet. B) Apply a clean occlusive dressing once daily and whenever soiled. C) Irrigate the wound with hydrogen peroxide once daily. D) Apply an antibiotic ointment on the surrounding skin with each dressing change.

Ans: Provide a high-calorie, high-protein diet. Feedback: Wound healing is highly dependent on adequate nutrition. The diet should be sufficiently high in calories and protein. Antibiotic ointments are not normally used on the skin surrounding a leg ulcer and occlusive dressings can exacerbate impaired blood flow. Hydrogen peroxide is not normally used because it can damage granulation tissue.

The nurse is caring for a patient who returned from the tropics a few weeks ago and who sought care with signs and symptoms of lymphedema. The nurse's plan of care should prioritize what nursing diagnosis? A) Risk for infection related to lymphedema B) Disturbed body image related to lymphedema C) Ineffective health maintenance related to lymphedema D) Risk for deficient fluid volume related to lymphedema

Ans: Risk for infection related to lymphedema Feedback: Lymphedema, which is caused by accumulation of lymph in the tissues, constitutes a significant risk for infection. The patient's body image is likely to be disturbed, and the nurse should address this, but infection is a more significant threat to the patient's physiological well-being. Lymphedema is unrelated to ineffective health maintenance and deficient fluid volume is not a significant risk.

A nurse is closely monitoring a patient who has recently been diagnosed with an abdominal aortic aneurysm. What assessment finding would signal an impending rupture of the patient's aneurysm? A) Sudden increase in blood pressure and a decrease in heart rate B) Cessation of pulsating in an aneurysm that has previously been pulsating visibly C) Sudden onset of severe back or abdominal pain D) New onset of hemoptysis

Ans: Sudden onset of severe back or abdominal pain Feedback: Signs of impending rupture include severe back or abdominal pain, which may be persistent or intermittent. Impending rupture is not typically signaled by increased blood pressure, bradycardia, cessation of pulsing, or hemoptysis.

The nurse is assessing a woman who is pregnant at 27 weeks' gestation. The patient is concerned about the recent emergence of varicose veins on the backs of her calves. What is the nurse's best response? A) Facilitate a referral to a vascular surgeon. B) Assess the patient's ankle-brachial index (ABI) and perform Doppler ultrasound testing. C) Encourage the patient to increase her activity level. D) Teach the patient that circulatory changes during pregnancy frequently cause varicose veins.

Ans: Teach the patient that circulatory changes during pregnancy frequently cause varicose veins. Feedback: Pregnancy may cause varicosities because of hormonal effects related to decreased venous outflow, increased pressure by the gravid uterus, and increased blood volume. In most cases, no intervention or referral is necessary. This finding is not an indication for ABI assessment and increased activity will not likely resolve the problem.

A nurse is reviewing the physiological factors that affect a patient's cardiovascular health and tissue oxygenation. What is the systemic arteriovenous oxygen difference? A) The average amount of oxygen removed by each organ in the body B) The amount of oxygen removed from the blood by the heart C) The amount of oxygen returning to the lungs via the pulmonary artery D) The amount of oxygen in aortic blood minus the amount of oxygen in the vena caval blood

Ans: The amount of oxygen in aortic blood minus the amount of oxygen in the vena caval blood Feedback: The average amount of oxygen removed collectively by all of the body tissues is about 25%. This means that the blood in the vena cava contains about 25% less oxygen than aortic blood. This is known as the systemic arteriovenous oxygen difference. The other answers do not apply.

Graduated compression stockings have been prescribed to treat a patient's venous insufficiency. What education should the nurse prioritize when introducing this intervention to the patient? A) The need to take anticoagulants concurrent with using compression stockings B) The need to wear the stockings on a "one day on, one day off" schedule C) The importance of wearing the stockings around the clock to ensure maximum benefit D) The importance of ensuring the stockings are applied evenly with no pressure points

Ans: The importance of ensuring the stockings are applied evenly with no pressure points Feedback: Any type of stocking can inadvertently become a tourniquet if applied incorrectly (i.e., rolled tightly at the top). In such instances, the stockings produce rather than prevent stasis. For ambulatory patients, graduated compression stockings are removed at night and reapplied before the legs are lowered from the bed to the floor in the morning. They are used daily, not on alternating days. Anticoagulants are not always indicated in patients who are using compression stockings.

The nurse is providing care for a patient who has just been diagnosed with peripheral arterial occlusive disease (PAD). What assessment finding is most consistent with this diagnosis? A) Numbness and tingling in the distal extremities B) Unequal peripheral pulses between extremities C) Visible clubbing of the fingers and toes D) Reddened extremities with muscle atrophy

Ans: Unequal peripheral pulses between extremities Feedback: PAD assessment may manifest as unequal pulses between extremities, with the affected leg cooler and paler than the unaffected leg. Intermittent claudication is far more common than sensations of numbness and tingling. Clubbing and muscle atrophy are not associated with PAD.

A patient presents to the clinic complaining of the inability to grasp objects with her right hand. The patient's right arm is cool and has a difference in blood pressure of more than 20 mm Hg compared with her left arm. The nurse should expect that the primary care provider may diagnose the woman with what health problem? A) Lymphedema B) Raynaud's phenomenon C) Upper extremity arterial occlusive disease D) Upper extremity VTE

Ans: Upper extremity arterial occlusive disease Feedback: The patient with upper extremity arterial occlusive disease typically complains of arm fatigue and pain with exercise (forearm claudication) and inability to hold or grasp objects (e.g., combing hair, placing objects on shelves above the head) and, occasionally, difficulty driving. Assessment findings include coolness and pallor of the affected extremity, decreased capillary refill, and a difference in arm blood pressures of more than 20 mm Hg. These symptoms are not closely associated with Raynaud's or lymphedema. The upper extremities are rare sites for VTE.

A nurse is assessing a new patient who is diagnosed with PAD. The nurse cannot feel the pulse in the patient's left foot. How should the nurse proceed with assessment? A) Have the primary care provider order a CT. B) Apply a tourniquet for 3 to 5 minutes and then reassess. C) Elevate the extremity and attempt to palpate the pulses. D) Use Doppler ultrasound to identify the pulses.

Ans: Use Doppler ultrasound to identify the pulses. Feedback: When pulses cannot be reliably palpated, a hand-held continuous wave (CW) Doppler ultrasound device may be used to hear (insonate) the blood flow in vessels. CT is not normally warranted and the application of a tourniquet poses health risks and will not aid assessment. Elevating the extremity would make palpation more difficult.

An occupational health nurse is providing an educational event and has been asked by an administrative worker about the risk of varicose veins. What should the nurse suggest as a proactive preventative measure for varicose veins? A) Sit with crossed legs for a few minutes each hour to promote relaxation. B) Walk for several minutes every hour to promote circulation. C) Elevate the legs when tired. D) Wear snug-fitting ankle socks to decrease edema.

Ans: Walk for several minutes every hour to promote circulation. Feedback: A proactive approach to preventing varicose veins would be to walk for several minutes every hour to promote circulation. Sitting with crossed legs may promote relaxation, but it is contraindicated for patients with, or at risk for, varicose veins. Elevating the legs only helps blood passively return to the heart and does not help maintain the competency of the valves in the veins. Wearing tight ankle socks is contraindicated for patients with, or at risk for, varicose veins; socks that are below the muscles of the calf do not promote venous return, the socks simply capture the blood and promote venous stasis.

The prevention of VTE is an important part of the nursing care of high-risk patients. When providing patient teaching for these high-risk patients, the nurse should advise lifestyle changes, including which of the following? Select all that apply. A) High-protein diet B) Weight loss C) Regular exercise D) Smoking cessation E) Calcium and vitamin D supplementation

Ans: Weight loss, Regular exercise, Smoking cessation Feedback: Patients at risk for VTE should be advised to make lifestyle changes, as appropriate, which may include weight loss, smoking cessation, and regular exercise. Increased protein intake and supplementation with vitamin D and calcium do not address the main risk factors for VTE.

7. A stroke victim is experiencing memory loss and impaired learning capacity. The nurse knows that brain damage has most likely occurred in which lobe? a. Frontal b. Occipital c. Parietal d. Temporal

a

ischemic stroke

a type of stroke that occurs when the flow of blood to the brain is blocked

The nurse observes a student nurse assigned to start oral feedings for a patient with an ischemic stroke. Which action by the student will require the nurse to intervene? Giving the patient 1 ounce of water to swallow Telling the patient to perform a chin tuck before swallowing Assisting the patient to sit in a chair before feeding the patient Assessing cranial nerves III, IV, and VI before attempting feeding

Assessing cranial nerves III, IV, and VI before attempting feeding Many patients after a stroke have dysphagia. The gag reflex and swallowing ability (cranial nerves IX and X) should be assessed before the first oral feeding. Cranial nerves III, IV, and VI are responsible for ocular movements. To assess swallowing ability, the nurse should elevate the head of the bed to an upright position (unless contraindicated) and give the patient a small amount of crushed ice or ice water to swallow. The patient should remain in a high Fowler's position, preferably in a chair with the head flexed forward, for the feeding and for 30 minutes following.

7. For a patient who is suspected of having a stroke, one of the most important pieces of information that the nurse can obtain is a. time of the patient's last meal. b. time at which stroke symptoms first appeared. c. patient's hypertension history and management. d. family history of stroke and other cardiovascular diseases.

B

During change of shift report, a nurse is told that a patient has an occluded left posterior cerebral artery. What finding should the nurse anticipate? a. Dysphasia b. Confusion c. Visual deficits d. Poor judgment

c. Visual deficits

A patient experiences a cerebral vascular accident (CVA) and is admitted to the hospital in a coma. What is the priority nursing care for this patient? A. Monitor vital signs B. Maintain an open airway C. Maintain fluid and electrolytes D. Monitor pupil response and equality

B. A patent airway is the priority because the airway may become occluded by the tongue in an unconscious patient. Monitoring vital signs is not the priority, although it is an important nursing function. Monitoring pupil response and equality and maintaining fluid and electrolytes are not the priority, although they are important nursing functions.

Which color of cerebrospinal fluid (CSF) may indicate subarachnoid hemorrhage in the patient? A. Hazy B. Yellow C. Brown D. Colorless

B. The yellow color of CSF can be attributed to the hemolysis of the red blood cells (RBCs), which leads to increased production of bilirubin. Other causes include subarachnoid hemorrhage, jaundice, increased CSF protein, hypercarotenemia, or hemoglobinemia. Hazy or unclear CSF is indicative of an elevated white blood cell (WBC) count due to infections. If the CSF has a brown color it is indicative of the presence of methemoglobin, indicating a previous meningeal hemorrhage. A colorless color indicates a normal finding.

2. The factor related to cerebral blood flow that most often determines the extent of cerebral damage from a stroke is the a. amount of cardiac output. b. O2 content of the blood. c. degree of collateral circulation. d. level of CO2 in the blood.

C

4. A patient is exhibiting word finding difficulty and weakness in his right arm. What area of the brain is most likely involved? a. brainstem. b. vertebral artery. c. left middle cerebral artery. d. right middle cerebral artery.

C

5. The nurse explains to the patient with a stroke who is scheduled for angiography that this test is used to determine the a. presence of increased ICP. b. site and size of the infarction. c. patency of the cerebral blood vessels. d. presence of blood in the cerebrospinal fluid

C

6. A patient experiencing TIAs is scheduled for a carotid endarterectomy. The nurse explains that this procedure is done to a. decrease cerebral edema. b. reduce the brain damage that occurs during a stroke in evolution. c. prevent a stroke by removing atherosclerotic plaques blocking cerebral blood flow. d. provide a circulatory bypass around thrombotic plaques obstructing cranial circulation.

C

8. Bladder training in a male patient who has urinary incontinence after a stroke includes a. limiting fluid intake. b. keeping a urinal in place at all times. c. assisting the patient to stand to void. d. catheterizing the patient every 4 hours.

C

A patient comes into the ED after hitting his head while playing basketball. He is alert and oriented. Which is the priority nursing intervention? A. Assess full ROM to determine extent of injuries B. Call for an immediate head CT C. Immobilize the patient's head and neck D. Open the airway with the head-tilt chin-lift maneuver

C. All patients with a head injury are treated as if a cervical spine injury is present until x-rays confirm their absence. ROM would be contraindicated at this time. The head CT would be prescribed next. The airway does not need to be opened because the patient appears alert and not in respiratory distress. In addition, the head-tilt chin-lift maneuver would not be used until the cervical spine injury is ruled out.

A patient experiences expressive aphasia as a result of a brain attack (cerebrovascular accident [CVA]). The patient's spouse asks whether the patient's speech will ever return. What is the best response by the nurse? A. "It should return in several months" B. "You will have to ask the primary HCP" C. "It is hard to say how much improvement will occur" D. "Unfortunately, your spouse will no longer be able to speak"

C. Recovery from aphasia is a continuous process; the amount of recovery cannot be predicted. Option A gives false reassurance; it may take a year or longer or may never return. B abdicates the nurse's responsibility; the HCP cannot predict return of function. Speech return is a continuous process; it may take a year or longer or may never return.

A nurse should plan to maintain a patient who has experienced a subarachnoid hemorrhage in what position? A. Supine B. On the unaffected side C. In bed with the HOB elevated D. With sandbags on either side of the head

C. With the head of the bed elevated, the force of gravity helps prevent additional ICP, which will intensify the ischemic manifestations of hemorrhage. The supine position will not facilitate drainage of cerebral fluid; this position promotes accumulation of fluid, which increases ICP. Lying on the unaffected side will not facilitate drainage of cerebral fluid; this position promotes accumulation of fluid, which increases ICP. Vomiting can occur with increased ICP, and placing sandbags to immobilize the head can result in aspiration.

1. Of the following patients, the nurse recognizes that the one with the highest risk for a stroke is a(n) a. obese 45yrold Native American. b. 35yrold Asian American woman who smokes. c. 32yrold white woman taking oral contraceptives. d. 65yrold African American man with hypertension.

D

3. Information provided by the patient that would help differentiate a hemorrhagic stroke from a thrombotic stroke includes a. sensory disturbance. b. a history of hypertension. c. presence of motor weakness. d. sudden onset of severe headache.

D

A patient with a brain attack (cerebrovascular accident) is admitted to the hospital. What is the priority nursing intervention for this patient? A. Changing position every two hours B. Keeping a serial record of the pulse C. Performing range-of-motion exercises D. Monitoring for increased intracranial pressure

D. Cerebral edema may occur with a brain attack, resulting in increased intracranial pressure. Although preventing pressure ulcers is important, it is not the priority. All vital signs are important, not just the pulse. Although maintaining joint mobility is important, it is not the priority; range-of-motion exercises may increase intracranial pressure and should be instituted in collaboration with the HCP.

In promoting health maintenance for prevention of strokes, the nurse understands that the highest risk for the most common type of stroke is present in a. African Americans b. women who smoke c. individuals with hypertension and diabetes d. those who are obese with high dietary fat intake

c. individuals with hypertension and diabetes

When providing care to the patient with an acute stroke, which duty can be delegated to the LPN/VN? Screen patient for tPA eligibility. Assess the patient's ability to swallow. Give scheduled anticoagulant medications. Place seizure precaution equipment in room.

Give scheduled anticoagulant medications. Assessment and screening are considered part of the registered nurse scope of practice. The LPN/VN can give PO or subcutaneous anticoagulant medications. Anticoagulant medications are considered high risk and should be double-checked with another LPN/VN or RN. The UAP can place equipment needed for seizure precautions in the room.

1. Which intervention should the nurse delegate to the PCA when caring for a patient following an acute stroke? a. assess the patient's neurologic status b. assess the patient's gag reflex before beginning feeding c. obtain the patients BP and HR d. teach the patient's caregivers strategies to minimize unilateral neglect

c. obtain the patients BP & HR

Which modifiable risk factor for stroke would be most important for the nurse to include when planning a community education program? Alcohol use Hypertension Hyperlipidemia Oral contraceptive use

Hypertension Hypertension is the single most important modifiable risk factor, but it is still often undetected and inadequately treated. The public is often more aware of hyperlipidemia and oral contraceptive use as risk factors for stroke. Alcohol is also a modifiable risk factor.

1. An appropriate food for a patient with a stroke who has mild dysphagia is a. fresh fruit b. Meats c. Scrambled eggs d. Broccoli

c. scrambled eggs

The nurse would expect what assessment finding in a patient admitted with a left-sided stroke? Impulsivity Impaired speech Left-side neglect Short attention span

Impaired speech Manifestations of left-sided brain damage include right hemiplegia, impaired speech/language, impaired right/left discrimination, and slow and cautious performance. Impulsivity, left-sided neglect, and short attention span are all manifestations of right-sided brain damage.

A CT scan of a patient's head reveals a hemorrhagic stroke. What is the priority nursing intervention in the emergency department? Maintaining the patient's airway Positioning to promote cerebral perfusion Controlling fluid and electrolyte imbalances Administering tissue plasminogen activator (tPA)

Maintaining the patient's airway Maintaining a patent airway is the priority in the acute care of a patient with a hemorrhagic stroke. It supersedes the importance of fluid and electrolyte imbalance and positioning. tPA is contraindicated in hemorrhagic stroke.

The neurologic functions that are affected by a stroke are primarily related to a. the amount of tissue area involved b. the rapidity of onset of symptoms c. the brain area perfused by the affected artery d. the presence or absence of collateral circulation

c. the brain area perfused by the affected artery

13. Eighty percent of hemorrhagic strokes are primarily caused by: a. an embolus. b. a cerebral thrombus. c. a brain tumor. d. uncontrolled hypertension.

d

6. The degree of neurologic damage that occurs with an ischemic stroke depends on the: a. location of the lesion. b. size of the area of inadequate perfusion. c. amount of collateral blood flow. d. combination of the above factors.

d

9. The most common motor dysfunction of a stroke is: a. ataxia. b. diplopia. c. dysphagia. d. hemiplegia.

d

A 74-yr-old man who has right-sided extremity paralysis related to a thrombotic stroke develops constipation. Which action should the nurse take first? Assist the patient to the bathroom every 2 hours. Provide incontinence briefs to wear during the day. Give a bisacodyl (Dulcolax) rectal suppository every day. Provide several servings daily of cooked fruits and vegetables.

Provide several servings daily of cooked fruits and vegetables. Patients after a stroke often have constipation. Dietary management includes the following: fluid intake of 2500 to 3000 mL daily, prune juice (120 mL) or stewed prunes daily, cooked fruit 3 times daily, cooked vegetables 3 times daily, and whole-grain cereal or bread 3 to 5 times daily. Patients with urinary incontinence should be assisted to the bathroom every 2 hours. Suppositories may be ordered for short-term management if the patient does not respond to increased fluid and fiber. Incontinence briefs are indicated as a short-term intervention for urinary incontinence.

The nurse is planning psychosocial support for the family of the patient who had a stroke. What factor will have the greatest impact on family coping? Specific patient neurologic deficits The patient's ability to communicate Rehabilitation potential of the patient Presence of complications of a stroke

Rehabilitation potential of the patient Although a patient's neurologic deficit might initially be severe, the ability of the patient to recover is most likely to positively impact the family's coping. Providing explanations and emotional support beginning in the acute phase through the rehabilitation phase will facilitate coping. Emphasizing successes will offer the most realistic hope for the patient's rehabilitation and helps maintain hope for the patient's future abilities.

Which sensory-perceptual deficit is associated with left-sided stroke (right hemiplegia)? Slow, fearful performance of tasks Overestimation of physical abilities Difficulty judging position and distance Impulsivity and impatience at performing tasks

Slow, fearful performance of tasks Patients with a left-sided stroke (right hemiplegia) are often slower in organization and performance of tasks and may have a fearful, anxious response to a stroke. Overconfidence, spatial disorientation, and impulsivity are more commonly associated with a right-sided stroke.

The patient with diabetes had a right-sided stroke. Which nursing intervention should the nurse plan to provide for this patient? Safety measures Patience with communication Mobility assistance on the right side Place food in the left side of patient's mouth.

Safety measures A patient with a right-sided stroke has spatial-perceptual deficits, tends to minimize problems, has a short attention span, is impulsive, and may have impaired judgment. Safety is the biggest concern for this patient. Hemiplegia occurs on the left side of this patient's body. The patient with a left-sided stroke has hemiplegia on the right, is more likely to have communication problems, and needs mobility assistance on the right side with food placed on the left side if the patient needs to be fed after a swallow evaluation has taken place.

A female patient presents to the emergency department reporting the most severe headache of her life. Which type of stroke should the nurse anticipate? TIA Embolic stroke Thrombotic stroke Subarachnoid hemorrhage

Subarachnoid hemorrhage Headache is common in a patient who has a subarachnoid hemorrhage or an intracerebral hemorrhage. A TIA is a transient loss of neurologic function usually without a headache. A headache may occur with an ischemic embolic stroke, but severe neurologic deficits are the initial symptoms. The ischemic thrombotic stroke manifestations progress in the first 72 hours as infarction and cerebral edema increase.

The nurse is discharging a patient admitted with a transient ischemic attack (TIA). For which medications might the nurse expect to provide discharge instructions? (Select all that apply.) Ticlopidine Clopidogrel Enoxaparin Dipyridamole Enteric-coated aspirin Tissue plasminogen activator (tPA)

Ticlopidine Clopidogrel Dipyridamole Enteric-coated aspirin Aspirin is the most frequently used antiplatelet agent. Other drugs to prevent clot formation include clopidogrel, dipyridamole, ticlopidine, combined dipyridamole and aspirin, and anticoagulant drugs such as oral warfarin. Tissue plasminogen activator is a fibrinolytic medication used to treat ischemic stroke, not prevent TIAs or strokes.

15. During the secondary assessment of the patient with a stroke, what should be included (select all that apply)? a. Gaze b. Sensation c Facial palsy d. Proprioception e. Current medications f. Distal motor function

a, b, c, d, f. The secondary assessment and ongoing neurologic monitoring include the gaze, facial palsy, distal motor function (part of the NIH Stroke Scale), sensation, proprioception, cognition, motor abilities, cerebellar function, and deep tendon reflexes. Current medications and history of hypertension are part of the primary assessment.

Which intervention is most appropriate when communicating with a patient with aphasia after a stroke? Present several thoughts at once so the patient can connect the ideas. Ask open-ended questions to give the patient the opportunity to speak. Use simple, short sentences with visual cues to enhance comprehension. Finish the patient's sentences to minimize frustration associated with slow speech.

Use simple, short sentences with visual cues to enhance comprehension. When communicating with a patient with aphasia, the nurse should present one thought or idea at a time. Ask questions that can be answered with a "yes," "no," or simple word. Use visual cues and allow time for the patient to comprehend and respond to conversation.

The nurse is discharging a patient admitted with a transient ischemic attack (TIA). For which medications might the nurse expect to provide discharge instructions (select all that apply.)? a. Ticlopidine b. Clopidogrel c. Enoxaparin d. Dipyridamole e. Enteric-coated aspirin f. Tissue plasminogen activator (tPA)

a, b, d, e Aspirin is the most frequently used antiplatelet agent. Other drugs to prevent clot formation include clopidogrel, dipyridamole, ticlopidine, combined dipyridamole and aspirin, and anticoagulant drugs such as oral warfarin. Tissue plasminogen activator is a fibrinolytic medication used to treat ischemic stroke, not prevent TIAs or strokes.

4. What are characteristics of a stroke caused by an intracerebral hemorrhage (select all that apply)? a. Carries a poor prognosis b. Caused by rupture of a vessel c. Strong association with hypertension d. Commonly occurs during or after sleep e. Creates a mass that compresses the brain

a, b, e. Strokes from intracerebral hemorrhage have a poor prognosis, are caused by the rupture of a blood vessel, are frequently atherosclerotic, and create a mass that compresses the brain. Thrombotic strokes are related to hypertension and occur during sleep or after sleep.

21. The rehabilitation nurse assesses the patient, caregiver, and family before planning the rehabilitation program for this patient. What needs to be included in this assessment (select all that apply)? a. Cognitive status of the family b. Patient resources and support c. Rehabilitation potential of the patient d. Body strength remaining after the stroke e. Physical status of body systems affected by the stroke f. Patient and caregiver expectations of the rehabilitation

a, d, e, f. The patient's rehabilitation potential and the expectations of the patient and caregiver related to the rehabilitation program will have a big impact on planning and carrying out the rehabilitation plan. The other things the rehabilitation nurse will assess are the physical status of all of the patient's body systems, presence of complications caused by the stroke or other chronic conditions, the cognitive status of the patient, and the family (including the patient and caregiver) resources and support.

After receiving change-of-shift report on the following four patients, which patient should the nurse see first? a. A 60-yr-old patient with right-sided weakness who has an infusion of tPA prescribed b. A 50-yr-old patient who has atrial fibrillation and a new order for warfarin (Coumadin) c. A 30-yr-old patient with a subarachnoid hemorrhage 2 days ago who has nimodipine scheduled d. A 40-yr-old patient who had a transient ischemic attack yesterday and has a dose of aspirin due

a. A 60-yr-old patient with right-sided weakness who has an infusion of tPA prescribed

A patient has a ruptured cerebral aneurysm and subarachnoid hemorrhage. Which intervention will the nurse include in the plan of care? a. Apply intermittent pneumatic compression stockings. b. Assist to dangle on edge of bed and assess for dizziness. c. Encourage patient to cough and deep breathe every 4 hours. d. Insert an oropharyngeal airway to prevent airway obstruction.

a. Apply intermittent pneumatic compression stockings.

A female patient who had a stroke 24 hours ago has expressive aphasia. What is an appropriate nursing intervention to help the patient communicate? a. Ask questions that the patient can answer with "yes" or "no." b. Develop a list of words that the patient can read and practice reciting. c. Have the patient practice her facial and tongue exercises with a mirror. d. Prevent embarrassing the patient by answering for her if she does not respond.

a. Ask questions that the patient can answer with "yes" or "no."

1. A patient is admitted to the hospital with a left hemiplegia. To determine the size and location and to ascertain whether a stroke is ischemic or hemorrhagic, the nurse anticipates that the health care provider will request a a. CT scan b. lumbar puncture c. cerebral arteriogram d. PET

a. CT Scan

The patient comes to the HCP office with pain, edema, and warm skin on her lower left leg. What test should the nurse expect to be ordered first? a. Duplex ultrasound b. Complete blood count (CBC) c. Magnetic resonance imaging (MRI) d. Computed venography (phlebogram)

a. Duplex ultrasound With manifestations of a VTE, the Duplex ultrasound is most widely used to diagnose VTE by identifying where a thrombus is found and its extent. D-dimer may also be drawn to determine if a VTE exists.

5. Which type of stroke is associated with endocardial disorders, has a rapid onset, and is unrelated to activity? a. Embolic b. Thrombotic c. Intracerebral hemorrhage d. Subarachnoid hemorrhage

a. Embolic strokes are associated with endocardial disorders such as atrial fibrillation, have a rapid onset, and are unrelated to activity

Which modifiable risk factor for stroke would be most important for the nurse to include when planning a community education program? a. Hypertension b. Hyperlipidemia c. Alcohol consumption d. Oral contraceptive use

a. Hypertension Hypertension is the single most important modifiable risk factor, but it is still often undetected and inadequately treated. The public is often more aware of hyperlipidemia and oral contraceptive use as risk factors for stroke. Alcohol is also a modifiable risk factor

A patient with a Type A dissection of the arch of the aorta has a decreased LOC and weak carotid pulses. What should the nurse anticipate that initial treatment of the patient will include? a. Immediate surgery to replace the torn area with a graft b. Administration of anticoagulants to prevent embolization c. Administration of packed red blood cells (RBCs) to replace blood loss d. Administration of antihypertensives to maintain a mean arterial pressure of 70 to 80 mm Hg

a. Immediate surgery to replace the torn area with a graft Immediate surgery is indicated when complications (such as occlusion of the carotid arteries) occur. Otherwise, initial treatment for aortic dissection involves a period of lowering the BP and myocardial contractility to diminish the pulsatile forces in the aorta. Anticoagulants would prolong and intensify the bleeding. Blood is given only if the dissection ruptures.

7. Indicate whether the following manifestations of a stroke are more likely to occur with right brain damage (R) or left brain damage (L). __________ a. Aphasia __________ b. Impaired judgment __________ c. Quick, impulsive behavior __________ d. Inability to remember words __________ e. Left homonymous hemianopsia __________ f. Neglect of the left side of the body __________ g. Hemiplegia of the right side of the body

a. L; b. R; c. R; d. L; e. R; f. R; g. L

A CT scan of a 68-yr-old male patient's head reveals that he has experienced a hemorrhagic stroke. What is the priority nursing intervention in the emergency department? a. Maintenance of the patient's airway b. Positioning to promote cerebral perfusion c. Control of fluid and electrolyte imbalances d. Administration of tissue plasminogen activator (tPA)

a. Maintenance of the patient's airway Maintenance of a patent airway is the priority in the acute care of a patient with a hemorrhagic stroke. It supersedes the importance of fluid and electrolyte imbalance and positioning. tPA is contraindicated in hemorrhagic stroke.

A 63-yr-old patient who began experiencing right arm and leg weakness is admitted to the emergency department. In which order will the nurse implement these actions included in the stroke protocol? a. Obtain CT scan without contrast. b. Infuse tissue plasminogen activator (tPA). c. Administer oxygen to keep O2 saturation >95%. d. Use National Institute of Health Stroke Scale to assess patient.

a. Obtain CT scan without contrast. b. Infuse tissue plasminogen activator (tPA). c. Administer oxygen to keep O2 saturation >95%. d. Use National Institute of Health Stroke Scale to assess patient.

Which surgical therapy for AAA is most likely to have the postoperative complication of renal injury? a. Open aneurysm repair (OAR) above the level of the renal arteries b. Excising only the weakened area of the artery and suturing the artery closed c. Bifurcated graft used in aneurysm repair when the AAA extends into the iliac arteries d. Endovascular graft procedure with an aortic graft inside the aneurysm via the femoral artery

a. Open aneurysm repair (OAR) above the level of the renal arteries With the aortic cross-clamping proximal and distal to the aneurysm, the open aneurysm repair (OAR) above the renal artery may cause kidney injury from lack of blood flow during the surgery. The saccular aneurysm may involve excising only the weakened area of the artery and suturing the artery closed, but this will not decrease renal blood flow. Renal blood flow will not be directly obstructed using the bifurcated graft or the minimally invasive endovascular aneurysm repair.

The nurse evaluates that treatment for the patient with an uncomplicated aortic dissection is successful when what happens? a. Pain is relieved. b. Surgical repair is completed. c. BP is increased to normal range. d. Renal output is maintained at 30 mL/hr.

a. Pain is relieved. Relief of pain is a sign that the dissection has stabilized, and it may be treated conservatively for an extended time with drugs that lower the BP and decrease myocardial contractility. Surgery is usually indicated for Type A aortic dissection or if complications occur.

A patient has atrial fibrillation and develops an acute arterial occlusion at the iliac artery bifurcation. What are the 6 Ps of acute arterial occlusion the nurse may assess in this patient that require immediate notification of the HCP?

a. Pain; b. pallor; c. pulselessness; d. paresthesia; e. paralysis; f. poikilothermia. The HCP requires immediate notification to begin immediate intervention to prevent tissue necrosis and gangrene.

What are characteristic of vasospastic disease (Raynaud's phenomenon) (select all that apply)? a. Predominant in young females b. May be associated with autoimmune disorders c. Precipitated by exposure to cold, caffeine, and tobacco d. Involves small cutaneous arteries of the fingers and toes e. Inflammation of small and medium-sized arteries and veins f. Episodes involve white, blue, and red color changes of fingertips

a. Predominant in young females b. May be associated with autoimmune disorders c. Precipitated by exposure to cold, caffeine, and tobacco d. Involves small cutaneous arteries of the fingers and toes f. Episodes involve white, blue, and red color changes of fingertips

In preparation for AAA repair surgery, what should the nurse include in patient teaching? a. Prepare the bowel on the night before surgery with laxatives or an enema. b. Use moisturizing soap to clean the skin three times the day before surgery. c. Eat a high-protein and high-carbohydrate breakfast to help with healing postoperatively. d. Take the prescribed oral antibiotic the morning of surgery before going to the operating room.

a. Prepare the bowel on the night before surgery with laxatives or an enema. Usually aortic surgery patients will have a bowel preparation, skin cleansing with an antimicrobial agent on the day before surgery, nothing by mouth after midnight on the day of the surgery, and IV antibiotics immediately before the incision is made. Patients with a history of CVD will receive a β-adrenergic blocker preoperatively to reduce morbidity and mortality. Each surgeon's protocol may be different.

Nurses in change-of-shift report are discussing the care of a patient with a stroke who has progressively increasing weakness and decreasing level of consciousness. Which patient problem do they determine has the highest priority for the patient? a. Risk for aspiration b. Impaired skin integrity c. Impaired physical mobility d. Disturbed sensory perception

a. Risk for aspiration

The patient with diabetes mellitus had a right-sided stroke. Which nursing intervention should the nurse plan to provide for this patient? a. Safety measures b. Patience with communication c. Mobility assistance on the right side d. Place food in the left side of patient's mouth.

a. Safety measures A patient with a right-sided stroke has spatial-perceptual deficits, tends to minimize problems, has a short attention span, is impulsive, and may have impaired judgment. Safety is the biggest concern for this patient. Hemiplegia occurs on the left side of this patient's body. The patient with a left-sided stroke has hemiplegia on the right, is more likely to have communication problems, and needs mobility assistance on the right side with food placed on the left side if the patient needs to be fed after a swallow evaluation has taken place.

The surgery area calls the transfer report for a 68-year-old, postmenopausal, female patient who smokes and takes hormone therapy. She is returning to the floor after a lengthy hip replacement surgery. Which factors present in this patient increase her risk for developing venous thromboembolism (VTE) related to Virchow's triad (select all that apply)? a. Smoking b. IV therapy c. Dehydration d. Estrogen therapy e. Orthopedic surgery f. Prolonged immobilization

a. Smoking b. IV therapy d. Estrogen therapy e. Orthopedic surgery f. Prolonged immobilization This patient is a smoker and on hormone therapy, both of which increase blood hypercoagulability. She will have an IV, and her fractured hip can cause VTE by damaging the venous endothelium. She is an older patient who has had an orthopedic surgery and may have experienced prolonged immobility postinjury and through her "lengthy hip replacement surgery," which contributes to venous stasis. These are representative of Virchow's triad in this patient. The other options are also related to Virchow's triad but not present in this patient via the transfer report.

14. During the acute phase of a stroke, the nurse assesses the patient's vital signs and neurologic status every 4 hours. What is a cardiovascular sign that the nurse would see as the body attempts to increase cerebral blood flow? a. Hypertension b. Fluid overload c. Cardiac dysrhythmias d. S3 and S4 heart sounds

a. The body responds to the vasospasm and decreased circulation to the brain that occurs with a stroke by increasing the BP, frequently resulting in hypertension. The other options are important cardiovascular factors to assess but they do not result from impaired cerebral blood flow.

A patient with carotid atherosclerosis asks the nurse to describe a carotid endarterectomy. Which response by the nurse is accurate? a. "The diseased portion of the artery is replaced with a synthetic graft." b. "The obstructing plaque is surgically removed from inside an artery in the neck." c. "A wire is threaded through an artery in the leg to the clots in the carotid artery, and clots are removed." d. "A catheter with a deflated balloon is positioned at the narrow area, and the balloon flattens the plaque."

b. "The obstructing plaque is surgically removed from inside an artery in the neck."

17. A newly admitted patient diagnosed with a right-sided brain stroke has a nursing diagnosis of disturbed visual sensory perception related to homonymous hemianopsia. Early in the care of the patient, what should the nurse do? a. Place objects on the right side within the patient's field of vision. b. Approach the patient from the left side to encourage the patient to turn the head. c. Place objects on the patient's left side to assess the patient's ability to compensate. d. Patch the affected eye to encourage the patient to turn the head to scan the environment.

a. The presence of homonymous hemianopia in a patient with right hemisphere brain damage causes a loss of vision in the left field bilaterally. Early in the care of the patient, objects should be placed on the right side of the patient in the field of vision and the nurse should approach the patient from the right side. Later in treatment, patients should be taught to turn the head and scan the environment and should be approached from the affected side to encourage head turning. Eye patches are used if patients have diplopia (double vision).

18. Four days following a stroke, a patient is to start oral fluids and feedings. Before feeding the patient, what should the nurse do first? a. Check the patient's gag reflex. b. Order a soft diet for the patient. c. Raise the head of the bed to a sitting position. d. Evaluate the patient's ability to swallow small amounts of crushed ice or ice water

a. Usually the speech therapist will have completed a swallowing study before a diet is ordered. The first step in providing oral feedings for a patient with a stroke is ensuring that the patient has an intact gag reflex because oral feedings will not be provided if the gag reflex is impaired. After placing the patient in an upright position, the nurse should then evaluate the patient's ability to swallow ice chips or ice water.

Which characteristics describe the anticoagulant warfarin (Coumadin) (select all that apply)? a. Vitamin K is the antidote b. Protamine sulfate is the antidote c. May be administered orally or subcutaneously d. May be administered intravenously or subcutaneously e. Monitor dosage using international normalized ratio (INR) f. Dosage monitored using activated partial thromboplastin time (aPTT)

a. Vitamin K is the antidote e. Monitor dosage using international normalized ratio (INR) Warfarin (Coumadin) is a vitamin K antagonist, so vitamin K is the antidote. It is monitored with the INR. It is only given orally. Protamine sulfate is the antidote for unfractionated heparin (UH) and LMWH. UH can be given subcutaneously or IV. It is monitored with activated partial thromboplastin time (aPTT). Hirudin derivatives are given IV or subcutaneously, do not have an antidote, and are monitored with aPTT. Argatroban, a synthetic thrombin inhibitor, is given only IV and is monitored with aPTT. Factor Xa inhibitor, fondaparinux (Arixtra), is given subcutaneously and does not need routine coagulation testing. Rivaroxaban (Xarelto), another factor Xa inhibitor, is given orally.

Four days following a stroke, a patient is to start oral fluids and feedings. Before feeding the patient, the nurse should first a. check the patient's gag reflex b. order a soft diet for the patient c. raise the head of the bed to sitting position d. evaluate the patient's ability to swallow small sips water

a. check the patient's gag reflex

The nurse teaches the patient with any venous disorder that the best way to prevent venous stasis and increase venous return is to a. take short walks. b. sit with the legs elevated. c. frequently rotate the ankles. d. continuously wear elastic compression stockings.

a. take short walks. During walking, the muscles of the legs continuously knead the veins, promoting movement of venous blood toward the heart. Walking is the best measure to prevent venous stasis and will be increased gradually. Elevating the legs will decrease edema. The other methods will help venous return, but they do not provide the benefit that ambulation does.

12. The most common side effect of t-PA is: a. an allergic reaction. b. bleeding. c. severe vomiting. d. a second stroke in 6 to 12 hours.

b

4. The majority of ischemic strokes have what type of origin? a. Cardiogenic embolic b. Cryptogenic c. Large artery thrombotic d. Small artery thrombotic

b

10. The initial diagnostic test for a stroke, usually performed in the emergency department, is a: a. 12-lead electrocardiogram. b. carotid ultrasound study. c. noncontrast computed tomogram. d. transcranial Doppler flow study.

c

The patient with VTE is receiving therapy with heparin and asks the nurse whether the drug will dissolve the clot in her leg. What is the best response by the nurse? a. "This drug will break up and dissolve the clot so that circulation in the vein can be restored." b. "The purpose of the heparin is to prevent growth of the clot or formation of new clots where the circulation is slowed." c. "Heparin won't dissolve the clot but it will inhibit the inflammation around the clot and delay the development of new clots." d. "The heparin will dilate the vein, preventing turbulence of blood flow around the clot that may cause it to break off and travel to the lungs."

b. "The purpose of the heparin is to prevent growth of the clot or formation of new clots where the circulation is slowed." Anticoagulant therapy with heparin or warfarin (Coumadin) does not dissolve clots but prevents propagation of the clot, development of new thrombi, and embolization. Clot lysis occurs naturally through the body's intrinsic fibrinolytic system or by the administration of thrombolytic agents

Which conditions characterize critical limb ischemia (select all that apply)? a. Cold feet b. Arterial leg ulcers c. Venous leg ulcers d. Gangrene of the leg e. No palpable peripheral pulses f. Rest pain lasting more than 2 weeks

b. Arterial leg ulcers d. Gangrene of the leg f. Rest pain lasting more than 2 weeks Arterial leg ulcers and/or gangrene of the leg caused by PAD and chronic ischemic rest pain lasting more than 2 weeks characterize critical limb ischemia. Optimal therapy is revascularization via bypass surgery.

A patient in the clinic reports a recent episode of dysphasia and left-sided weakness at home that resolved after 2 hours. What topic should the nurse anticipate teaching the patient? a. tPA b. Aspirin c. Warfarin d. Nimodipine

b. Aspirin

Several weeks after a stroke, a 50-yr-old male patient has impaired awareness of bladder fullness, resulting in urinary incontinence. Which nursing intervention should be planned to begin an effective bladder training program? a. Limit fluid intake to 1200 mL daily to reduce urine volume. b. Assist the patient onto the bedside commode every 2 hours. c. Perform intermittent catheterization after each voiding to check for residual urine. d. Use an external "condom" catheter to protect the skin and prevent embarrassment.

b. Assist the patient onto the bedside commode every 2 hours.

During the patient's acute postoperative period following repair of an AAA, the nurse should ensure that which goal is achieved? a. Hypothermia is maintained to decrease oxygen need. b. BP and all peripheral pulses are evaluated at least every hour. c. IV fluids are administered at a rate to maintain urine output of 100 mL/hr. d. The patient's BP is kept lower than baseline to prevent leaking at the incision line.

b. BP and all peripheral pulses are evaluated at least every hour. The BP and peripheral pulses are evaluated every hour in the acute postoperative period to ensure that BP is adequate to maintain graft patency and that extremities are being perfused. BP is kept within normal range. If BP is too low, thrombosis of the graft may occur; if it is too high, it may cause leaking or rupture at the suture line. Hypothermia is induced during surgery, but the patient is rewarmed as soon as surgery is over. Fluid replacement to maintain urine output at 100 mL/hr would increase the BP too much and only 30 mL/hr of urine is needed to show adequate renal perfusion.

What will the nurse tell the patient who has cerebral atherosclerosis about taking clopidogrel (Plavix)? a. Monitor and record the blood pressure daily. b. Call the health care provider if stools are tarry. c. Clopidogrel will dissolve clots in the cerebral arteries. d. Clopidogrel will reduce cerebral artery plaque formation.

b. Call the health care provider if stools are tarry.

A 70-yr-old female patient with left-sided hemiparesis arrives by ambulance to the emergency department. Which action should the nurse take first? a. Take the patient's blood pressure. b. Check the respiratory rate and effort. c. Assess the Glasgow Coma Scale score. d. Send the patient for a computed tomography (CT) scan.

b. Check the respiratory rate and effort.

The patient is diagnosed with a superficial vein thrombosis (SVT). Which characteristic should the nurse know about SVT? a. Embolization to lungs may result in death. b. Clot may extend to deeper veins if untreated. c. Vein is tender to pressure and there is edema. d. Typically found in the iliac, inferior, or superior vena cava.

b. Clot may extend to deeper veins if untreated. If left untreated, a superficial vein thrombosis (SVT) may extend to deeper veins and VTE may occur. VTE may embolize to the lungs and have tenderness to pressure and edema. SVTs usually occur in superficial leg veins and have tenderness, itchiness, redness, warmth, pain, inflammation, and induration along the course of the superficial vein

Priority Decision: A patient who is postoperative following repair of an AAA has been receiving IV fluids at 125 mL/hr continuously for the last 12 hours. Urine output for the last 4 hours has been 60 mL, 42 mL, 28 mL, and 20 mL, respectively. What is the priority action that the nurse should take? a. Monitor for a couple more hours. b. Contact the physician and report the decrease in urine output. c. Send blood for electrolytes, blood urea nitrogen (BUN), and creatinine. d. Decrease the rate of infusion to prevent blood leakage at the suture line.

b. Contact the HCP and report the decrease in urine output. The decreasing urine output is evidence that either the patient needs volume replacement or there is reduced renal blood flow. The HCP will want to be notified as soon as possible of this change in condition and will request results of daily blood urea nitrogen (BUN) and serum creatinine levels. The other options are incorrect.

A patient with a small AAA is not a good surgical candidate. What should the nurse teach the patient is one of the best ways to prevent expansion of the lesion? a. Avoid strenuous physical exertion. b. Control hypertension with prescribed therapy. c. Comply with prescribed anticoagulant therapy. d. Maintain a low-calcium diet to prevent calcification of the vessel.

b. Control hypertension with prescribed therapy. Increased systolic BP (SBP) continually puts pressure on the diseased area of the artery, promoting its expansion. Small aneurysms can be treated by decreasing BP, modifying atherosclerosis risk factors, and monitoring the size of the aneurysm. Anticoagulants are used during surgical treatment of aneurysms, but physical activity is not known to increase their size. Calcium intake is not related to calcification in arteries.

14. A classic diagnostic system of hemorrhagic stroke is the patient's compliant of: a. numbness of an arm or leg. b. double vision. c. severe headache. d. dizziness and tinnitus.

c

8. The initial mortality rate for a stroke can be as high as: a. 10%. b. 20%. c. 30%. d. 50%.

c

22. What is an appropriate nursing intervention to promote communication during rehabilitation of the patient with aphasia? a. Use gestures, pictures, and music to stimulate patient responses. b. Talk about activities of daily living (ADLs) that are familiar to the patient. c. Structure statements so that the patient does not have to respond verbally. d. Use flashcards with simple words and pictures to promote recall of language.

b. During rehabilitation, the patient with aphasia needs frequent, meaningful verbal stimulation that has relevance for him or her. Conversation by the nurse and family should address activities of daily living (ADLs) that are familiar to the patient. Gestures, pictures, and simple statements are more appropriate in the acute phase, when patients may be overwhelmed by verbal stimuli. Flashcards are often perceived by the patient as childish and meaningless. Not responding verbally does not promote communication

The nurse would expect to find what clinical manifestation in a patient admitted with a left-sided stroke? a. Impulsivity b. Impaired speech c. Left-side neglect d. Short attention span

b. Impaired speech Clinical manifestations of left-sided brain damage include right hemiplegia, impaired speech/language, impaired right/left discrimination, and slow and cautious performance. Impulsivity, left-sided neglect, and short attention span are all manifestations of right-sided brain damage

When teaching the patient with PAD about modifying risk factors associated with the condition, what should the nurse emphasize? a. Amputation is the ultimate outcome if the patient does not alter lifestyle behaviors. b. Modifications will reduce the risk of other atherosclerotic conditions such as stroke. c. Risk-reducing behaviors initiated after angioplasty can stop the progression of the disease. d. Maintenance of normal body weight is the most important factor in controlling arterial disease

b. Modifications will reduce the risk of other atherosclerotic conditions such as stroke. PAD occurs as a result of atherosclerosis and the risk factors are the same as for other diseases associated with atherosclerosis, such as CAD, cerebrovascular disease, and aneurysms. Major risk factors are tobacco use, hyperlipidemia, elevated C-reactive protein, diabetes, obesity, and uncontrolled hypertension. The risk for amputation is high in patients with severe occlusive disease, but this is not the best approach to encourage patients to make lifestyle modifications.

During preoperative preparation of the patient scheduled for an AAA, why should the nurse establish baseline data for the patient? a. All physiologic processes will be altered postoperatively. b. The cause of the aneurysm is a systemic vascular disease. c. Surgery will be canceled if any physiologic function is not normal. d. BP and HR will be maintained well below baseline levels during the postoperative period.

b. The cause of the aneurysm is a systemic vascular disease Because atherosclerosis is a systemic disease, the patient with an AAA is likely to have cardiac, pulmonary, cerebral, or lower extremity vascular problems that should be noted and monitored throughout the perioperative period. Postoperatively, the BP is balanced: high enough to keep adequate flow through the artery to prevent thrombosis but low enough to prevent bleeding at the surgical site..

The nurse is caring for a patient who has just returned after having left carotid artery angioplasty and stenting. Which assessment information is of most concern to the nurse? a. The pulse rate is 102 beats/min. b. The patient has difficulty speaking. c. The blood pressure is 144/86 mmHg. d. There are fine crackles at the lung bases.

b. The patient has difficulty speaking.

Which information about the patient who had a subarachnoid hemorrhage is most important to communicate to the health care provider? a. The patient reports having a stiff neck. b. The patient's blood pressure (BP) is 90/50 mm Hg. c. The patient reports a severe and unrelenting headache. d. The cerebrospinal fluid (CSF) report shows red blood cells (RBCs).

b. The patient's blood pressure (BP) is 90/50 mm Hg.

What are characteristics of PAD (select all that apply)? a. Pruritus b. Thickened, brittle nails c. Dull ache in calf or thigh d. Decreased peripheral pulses e. Pallor on elevation of the legs f. Ulcers over bony prominences on toes and feet

b. Thickened, brittle nails d. Decreased peripheral pulses e. Pallor on elevation of the legs f. Ulcers over bony prominences on toes and feet PAD is manifested as thick, brittle nails; decreased peripheral pulses; pallor when the legs are elevated; ulcers over bony prominences on the toes and feet; and paresthesia. The other options are characteristic of venous disease and paresthesia could occur with venous thromboembolism (VTE).

A patient with PAD has a nursing diagnosis of ineffective peripheral tissue perfusion. What should be included in the teaching plan for this patient (select all that apply)? a. Apply cold compresses when the legs become swollen. b. wear protective footwear and avoid hot or cold extremes. c. Walk at least 30 minutes per day, at least 3 times per week. d. Use nicotine replacement therapy as a substitute for smoking. e. Inspect lower extremities for pulses, temperature, and any injury.

b. wear protective footwear and avoid hot or cold extremes. c. Walk at least 30 minutes per day, at least 3 times per week. e. Inspect lower extremities for pulses, temperature, and any injury. Protecting feet and legs from injury is important. Walking exercise increases oxygen extraction in the legs and improves skeletal muscle metabolism. The patient with PAD should walk at least 30 minutes a day, at least 3 times per week. Exercise should be stopped when pain occurs and resumed when the pain subsides. The lower extremities should be assessed at regular intervals for changes. Cold compresses and nicotine in all forms causes vasoconstriction and should be avoided.

Following teaching about medications for PAD, the nurse determines that additional instruction is necessary when the patient says, a) "I should take one aspirin a day to prevent clotting in my legs." b) "The lisinopril I use for my BP may help me walk further without pain." c) "I will need to have frequent blood tests to evaluate the effect of the Trental I will be taking." d) "Pletal should help me be able to increase my walking distance and speed and help prevent pain in the legs."

c) "I will need to have frequent blood tests to evaluate the effect of the Trental I will be taking." Pentoxifylline allows blood cells to pass through small vessels, but there are no blood tests related to it. Warfarin (Coumadin), which needs international normalized ration (INR) blood tests, is not recommended for prevention of cardiovascular disease (CVD) events in patients with PAD. All the other statements are correct in relation to treatment of PAD.

10. A carotid endarterectomy is being considered as treatment for a patient who has had several TIAs. What should the nurse explain to the patient about this surgery? a. It involves intracranial surgery to join a superficial extracranial artery to an intracranial artery. b. It is used to restore blood circulation to the brain following an obstruction of a cerebral artery. c. It involves removing an atherosclerotic plaque in the carotid artery to prevent an impending stroke. d. It is used to open a stenosis in a carotid artery with a balloon and stent to restore cerebral circulation.

c. A carotid endarterectomy is the removal of an atherosclerotic plaque in the carotid arteries that may impair circulation enough to cause a stroke. The other procedures described may also be used to prevent strokes. An extracranial-intracranial bypass involves cranial surgery to bypass a sclerotic intracranial artery. Stenting may improve circulation in the brain. A percutaneous transluminal angioplasty uses a balloon to compress stenotic areas in the carotid and vertebrobasilar arteries and often includes inserting a stent to hold the artery open.

Priority Decision: Following an ascending aortic aneurysm repair, what is an important finding that the nurse should report immediately to the health care provider? a. Shallow respirations and poor coughing b. Decreased drainage from the chest tubes c. A change in level of consciousness (LOC) and inability to speak d. Lower extremity pulses that are decreased from the preoperative baseline

c. A change in level of consciousness (LOC) and inability to speak During repair of an ascending aortic aneurysm, the blood supply to the carotid arteries may be interrupted, leading to neurologic complications manifested by a decreased level of consciousness (LOC) and altered pupil responses to light as well as changes in facial symmetry, speech, upper extremity movement, and hand grasp quality. The thorax is opened for ascending aortic surgery, and shallow breathing, poor cough, and decreasing chest drainage are expected. Lower limb pulses may normally be decreased or absent for a short time following surgery.

During care of the patient following femoral bypass graft surgery, the nurse immediately notifies the health care provider if the patient experiences... a. fever and redness at the incision site b. 2+ edema of the extremity and pain at the incision site c. A loss of palpable pulses and numbness and tingling of the feet d. increasing ankle-brachial indices and serous drainage from the incision

c. A loss of palpable pulses and numbness and tingling of the feet Loss of palpable pulses, numbness and tingling of the extremity, extremity pallor, cyanosis, or cold are indications of occlusion of the bypass graft and need immediate medical attention. Pain, redness, and serous drainage at the incision site are expected postoperatively. Ankle brachial index measurements are not recommended because of increased risk for graft thrombosis, but this would decrease with occlusion

When providing care to the patient with an acute stroke, which duty can be delegated to the LPN/LVN? a. Screen patient for tPA eligibility. b. Assess the patient's ability to swallow. c. Administer scheduled anticoagulant medications. d. Place equipment needed for seizure precautions in room

c. Administer scheduled anticoagulant medications. Assessment and screening are considered part of the registered nurse scope of practice. The LPN/LVN can administer PO or subcutaneous anticoagulant medications. Anticoagulant medications are considered high risk and should be double checked with another LPN/LVN or RN. The UAP can place equipment needed for seizure precautions in the room

The nurse is caring for a patient who has been experiencing stroke symptoms for 60 minutes. Which action can the nurse delegate to a licensed practical/vocational nurse (LPN/VN)? a. Assess the patient's gag and cough reflexes. b. Determine when the stroke symptoms began. c. Administer the prescribed short-acting insulin. d. Infuse the prescribed IV metoprolol (Lopressor).

c. Administer the prescribed short-acting insulin.

1. What is a risk factor of stroke that you CANNOT change? a. Weight b. High Blood Pressure c. Age d. Tobacco Use

c. Age

A patient will attempt oral feedings for the first time after having a stroke. After assessing the gag reflex, what action should the nurse take? a. Order a varied pureed diet. b. Assess the patient's appetite. c. Assist the patient into a chair. d. Offer the patient a sip of juice.

c. Assist the patient into a chair.

A left-handed patient with left-sided hemiplegia has difficulty feeding himself. Which intervention should the nurse include in the plan of care? a. Provide a wide variety of food choices. b. Provide oral care before and after meals. c. Assist the patient to eat with the right hand. d. Teach the patient the "chin-tuck" technique.

c. Assist the patient to eat with the right hand.

A female patient has left-sided hemiplegia after an ischemic stroke 4 days earlier. How should the nurse promote skin integrity? a. Position the patient on her weak side the majority of the time. b. Alternate the patient's positioning between supine and side-lying. c. Avoid the use of pillows in order to promote independence in positioning. d. Establish a schedule for the massage of areas where skin breakdown emerges.

c. Avoid the use of pillows in order to promote independence in positioning. A position change schedule should be established for stroke patients. An example is side-back-side, with a maximum duration of 2 hours for any position. The patient should be positioned on the weak or paralyzed side for only 30 minutes. Pillows may be used to facilitate positioning. Areas of skin breakdown should never be massaged.

A patient with left-sided weakness that started 60 minutes earlier is admitted to the emergency department and diagnostic tests are ordered. Which test should be done first? a. Complete blood count (CBC) b. Chest radiograph (chest x-ray) c. Computed tomography (CT) scan d. 12-Lead electrocardiogram (ECG)

c. Computed tomography (CT) scan

What is the most important measure in the treatment of venous leg ulcers? a. Elevation of the affected leg b. Application of topical antibiotics c. Graduated compression stockings d. Application of moist to dry dressings

c. Graduated compression stockings Although leg elevation, moist dressings, and systemic antibiotics are useful in treatment of venous stasis ulcers, the most important factor is compression, which minimizes venous stasis, venous hypertension, and edema and prevents recurrence. Compression may be applied with various methods including stockings, elastic bandages or wraps, or a Velcro wrap, among others.

9. A patient is admitted to the hospital with a left hemiplegia. To determine the size and location and to ascertain whether a stroke is ischemic or hemorrhagic, the nurse anticipates that the health care provider will request a a. lumbar puncture. b. cerebral arteriogram. c. magnetic resonance imaging (MRI). d. computed tomography (CT) scan with contrast.

c. MRI could be used to rapidly distinguish between ischemic and hemorrhagic stroke and determine the size and location of the lesion. A noncontrast CT scan could also be used. Lumbar punctures are not performed routinely because of the chance of increased intracranial pressure causing herniation. Cerebral arteriograms are invasive and may dislodge an embolism or cause further hemorrhage. They are performed only when no other test can provide the needed information.

26. Delegation Decision: Which intervention should the nurse delegate to the licensed practical nurse (LPN) when caring for a patient following an acute stroke? a. Assess the patient's neurologic status. b. Assess the patient's gag reflex before beginning feeding. c. Administer ordered antihypertensives and platelet inhibitors. d. Teach the patient's caregivers strategies to minimize unilateral neglect.

c. Medication administration is within the scope of practice for a licensed practical nurse (LPN). Assessment and teaching are within the scope of practice for the RN.

What topic should the nurse anticipate teaching a patient who had a brief episode of tinnitus, diplopia, and dysarthria with no residual effects? a. Cerebral aneurysm clipping b. Heparin intravenous infusion c. Oral low-dose aspirin therapy d. Tissue plasminogen activator (tPA)

c. Oral low-dose aspirin therapy

24. A patient with a stroke has a right-sided hemiplegia. What does the nurse teach the family to prepare them to cope with the behavior changes seen with this type of stroke? a. Ignore undesirable behaviors manifested by the patient. b. Provide directions to the patient verbally in small steps. c. Distract the patient from inappropriate emotional responses. d. Supervise all activities before allowing the patient to pursue them independently.

c. Patients with left-brain damage from stroke often experience emotional lability, inappropriate emotional responses, mood swings, and uncontrolled tears or laughter disproportionate to or out of context with the situation. The behavior is upsetting and embarrassing to both the patient and the family and the patient should be distracted to minimize its presence. Maintaining a calm environment and avoiding shaming or scolding the patient is important. Patients with right-brain damage often have impulsive, rapid behavior that requires supervision and direction.

Which intervention should the nurse include in the plan of care for a patient with new right-sided homonymous hemianopsia after a stroke? a. Apply an eye patch to the right eye. b. Approach the patient from the right side. c. Place needed objects on the patient's left side. d. Teach the patient that the left visual deficit will resolve.

c. Place needed objects on the patient's left side.

A surgical repair is planned for a patient who has a 5.5-cm abdominal aortic aneurysm (AAA). On physical assessment of the patient, what should the nurse expect to find? a. Hoarseness and dysphagia b. Severe back pain with flank ecchymosis c. Presence of a bruit in the periumbilical area d. Weakness in the lower extremities progressing to paraplegia

c. Presence of a bruit in the periumbilical area Although most abdominal aortic aneurysms (AAAs) are asymptomatic, on physical examination a pulsatile mass in the periumbilical area slightly to the left of the midline may be detected and bruits may be audible with a stethoscope placed over the aneurysm. Hoarseness and dysphagia may occur with aneurysms of the ascending aorta and the aortic arch. Severe back pain with flank ecchymosis is usually present on rupture of an AAA and neurovascular loss in the lower extremities may occur from pressure of a thoracic aneurysm.

The home health nurse is caring for an 81-yr-old who had a stroke 2 months ago. Based on patient information shown in the accompanying figure, which action should the nurse take? a. Teach about preventing hypoglycemia. b. Begin processes to obtain a wheelchair. c. Provide support to the spouse caregiver. d. Remind the patient to take prescribed medications.

c. Provide support to the spouse caregiver.

8. The patient has a lack of comprehension of both verbal and written language. Which type of communication difficulty does this patient have? a. Dysarthria b. Fluent dysphasia c. Receptive aphasia d. Expressive aphasia

c. Receptive aphasia is the lack of comprehension of both verbal and written language. Dysarthria is disturbance in muscular control of speech. In fluent dysphasia speech is present but contains little meaningful communication. Expressive aphasia is the loss of the production of language.

The nurse is planning psychosocial support for the family of the patient who suffered a stroke. What factor will have the greatest impact on family coping? a. Specific patient neurologic deficits b. The patient's ability to communicate c. Rehabilitation potential of the patient d. Presence of complications of a stroke

c. Rehabilitation potential of the patient Although a patient's neurologic deficit might initially be severe, the ability of the patient to recover is most likely to positively impact the family's coping. Providing explanations and emotional support beginning in the acute phase through the rehabilitation phase will facilitate coping. Emphasizing successes will offer the most realistic hope for the patient's rehabilitation and helps maintain hope for the patient's future abilities.

Delegation Decision: The nursing care area is very busy with new surgical patients. Which care could the RN delegate to the unlicensed assistive personnel (UAP) for a patient with VTE? a. Assess the patient's use of herbs. b. Measure the patient for elastic compression stockings. c. Remind the patient to flex and extend the legs and feet every 2 hours d. Teach the patient to call emergency medical services (EMS) with signs of pulmonary embolus

c. Remind the patient to flex and extend the legs and feet every 2 hours The RN could delegate to the UAP the task to remind the patient to flex and extend the legs and feet every 2 hours while in bed. Measuring for elastic compression stockings may be delegated to the LPN. The RN must assess and teach the patient.

During the nursing assessment of the patient with a distal descending aortic dissection, what should the nurse expect the patient to manifest? a. Altered LOC with dizziness and weak carotid pulses b. A cardiac murmur characteristic of aortic valve insufficiency c. Severe "ripping" back or abdominal pain with decreasing urine output d. Severe hypertension and orthopnea and dyspnea of pulmonary edema

c. Severe "ripping" back or abdominal pain with decreasing urine output A Type B aortic dissection involves the distal descending aorta and is usually characterized by a sudden, severe, tearing pain in the back. As it progresses down the aorta, the kidneys, abdominal organs, and lower extremities may begin to show evidence of ischemia. Type A aortic dissections of the ascending aorta and aortic arch may affect the heart and circulation to the head, with the development of cerebral ischemia, murmurs, ventricular failure, and pulmonary edema.

Which sensory-perceptual deficit is associated with left-sided stroke (right hemiplegia)? a. Overestimation of physical abilities b. Difficulty judging position and distance c. Slow and possibly fearful performance of tasks d. Impulsivity and impatience at performing tasks

c. Slow and possibly fearful performance of tasks Patients with a left-sided stroke (right hemiplegia) commonly are slower in organization and performance of tasks and may have a fearful, anxious response to a stroke. Overconfidence, spatial disorientation, and impulsivity are more commonly associated with a right-sided stroke.

19. What is an appropriate food for a patient with a stroke who has mild dysphagia? a. Fruit juices b. Pureed meat c. Scrambled eggs d. Fortified milkshakes

c. Soft foods that provide enough texture, flavor, and bulk to stimulate swallowing should be used for the patient with dysphagia. Thin liquids are difficult to swallow and patients may not be able to control them in the mouth. Pureed foods are often too bland and too smooth and milk products should be avoided because they tend to increase the viscosity of mucus and increase salivation.

A patient admitted with possible stroke has been aphasic for 3 hours and has a current blood pressure (BP) of 174/94 mm Hg. Which order by the health care provider should the nurse question? a. Keep head of bed elevated at least 30 degrees. b. Infuse normal saline intravenously at 75 mL/hr. c. Start a labetalol drip to keep BP less than 140/90 mm Hg. d. Begin tissue plasminogen activator (tPA) intravenously per protocol.

c. Start a labetalol drip to keep BP less than 140/90 mm Hg.

11. The incidence of ischemic stroke in patients with TIAs and other risk factors is reduced with the administration of which medication? a. Furosemide (Lasix) b. Lovastatin (Mevacor) c. Daily low-dose aspirin d. Nimodipine (Nimotop)

c. The administration of antiplatelet agents, such as aspirin, ticlopidine (Ticlid), clopidogrel (Plavix), dipyridamole (Persantine), and combined dipyridamole and aspirin (Aggrenox), reduces the incidence of stroke in those at risk. Anticoagulants are also used for prevention of embolic strokes but increase the risk for hemorrhage. Diuretics are not indicated for stroke prevention other than for their role in controlling blood pressure (BP) and antilipidemic agents have not been found to have a significant effect on stroke prevention. The calcium channel blocker nimodipine is used in patients with subarachnoid hemorrhage to decrease the effects of vasospasm and minimize tissue damage

2. A thrombus that develops in a cerebral artery does not always cause a loss of neurologic function because a. the body can dissolve atherosclerotic plaques as they form. b. some tissues of the brain do not require constant blood supply to prevent damage. c. circulation via the circle of Willis may provide blood supply to the affected area of the brain. d. neurologic deficits occur only when major arteries are occluded by thrombus formation around atherosclerotic plaque.

c. The communication between the anterior and posterior cerebral circulation in the circle of Willis provides a collateral circulation, which may maintain circulation to an area of the brain if its original blood supply is obstructed. All areas of the brain require constant blood supply and atherosclerotic plaques are not readily reversed. Neurologic deficits can result from ischemia caused by many factors.

1. In promoting health maintenance for prevention of strokes, the nurse understands that the highest risk for the most common type of stroke is present in which people? a. African Americans b. Women who smoke c. Individuals with hypertension and diabetes d. Those who are obese with high dietary fat intake

c. The highest risk factors for thrombotic stroke are hypertension and diabetes. African Americans have a higher risk for stroke than do white persons, probably because they have a greater incidence of hypertension, diabetes mellitus, and obesity. Factors such as diet high in saturated fats and cholesterol, cigarette smoking, metabolic syndrome, sedentary lifestyle, and excessive alcohol use are also risk factors but carry less risk than hypertension.

A patient is being admitted with a possible stroke. Which information from the assessment indicates that the nurse should consult with the health care provider before giving a prescribed dose of aspirin? a. The patient has dysphasia. b. The patient has atrial fibrillation. c. The patient reports that symptoms began with a severe headache. d. The patient has a history of brief episodes of right-sided hemiplegia.

c. The patient reports that symptoms began with a severe headache.

Which stroke risk factor for a 48-yr-old male patient in the clinic is most important for the nurse to address? a. The patient is 25 pounds above the ideal weight. b. The patient drinks a glass of red wine with dinner daily. c. The patient's usual blood pressure (BP) is 170/94 mm Hg. d. The patient works at a desk and relaxes by watching television.

c. The patient's usual blood pressure (BP) is 170/94 mm Hg.

23. A patient with a right hemisphere stroke has a nursing diagnosis of unilateral neglect related to sensory-perceptual deficits. During the patient's rehabilitation, what nursing intervention is important for the nurse to do? a. Avoid positioning the patient on the affected side. b. Place all objects for care on the patient's unaffected side. c. Teach the patient to care consciously for the affected side. d. Protect the affected side from injury with pillows and supports.

c. Unilateral neglect, or neglect syndrome, occurs when the patient with a stroke is unaware of the affected side of the body, which puts the patient at risk for injury. During the acute phase, the affected side is cared for by the nurse with positioning and support but during rehabilitation the patient is taught to care consciously for and attend to the affected side of the body to protect it from injury. Patients may be positioned on the affected side for up to 30 minutes.

When obtaining a health history from a 72 year old man with peripheral arterial disease (PAD) of the lower extremities, the nurse asks about a history of related conditions, including? a) Venous thrombosis b) Venous stasis ulcers c) Pulmonary embolism d) Coronary artery disease

d) Coronary artery disease Regardless of the location, atherosclerosis is responsible for peripheral arterial disease (PAD) and is related to other cardiovascular disease and its risk factors, such as coronary artery disease (CAD) and carotid artery disease. Venous thrombosis, venous stasis ulcers, and pulmonary embolism are diseases of the veins and are not related to atherosclerosis.

The nurse is teaching a senior citizen's group about signs and symptoms of a stroke. Which statement by the nurse would provide accurate information? a. "Take the person to the hospital if a headache lasts for more than 24 hours." b. "Stroke symptoms usually start when the person is awake and physically active." c. "A person with a transient ischemic attack has mild symptoms that will go away." d. "Call 911 immediately if a person develops slurred speech or difficulty speaking.

d. "Call 911 immediately if a person develops slurred speech or difficulty speaking. Medical assistance should be obtained immediately for someone with signs and symptoms of a stroke such as sudden numbness; weakness; paralysis of the face, arm, or leg (especially on one side of the body); sudden confusion; trouble speaking or understanding; slurred speech; sudden trouble seeing in one or both eyes; sudden trouble walking; dizziness; loss of balance or coordination; or a sudden, severe headache with no known cause. A person with signs and symptoms of a transient ischemic attack should seek medical attention immediately because it is unknown if the symptoms will resolve or persist and progress to a stroke. Onset of signs and symptoms of a stroke vary depending on the type. Onset of an ischemic thrombotic stroke usually occurs at rest. Onset of an ischemic embolic stroke is not related to rest or activity, and onset of a hemorrhagic stroke usually occurs with activity

Following discharge teaching with a male patient with an AAA repair, the nurse determines that further instruction is needed when the patient makes which statement? a. "I should avoid heavy lifting for 6 weeks." b. "I may have some sexual dysfunction as a result of the surgery." c. "I should maintain a low-fat and low-cholesterol diet to help keep the new graft open." d. "I should take the pulses in my extremities and let the doctor know if they get too fast or too slow."

d. "I should take the pulses in my extremities and let the doctor know if they get too fast or too slow." Patients are taught to palpate peripheral pulses to identify changes in their quality or strength, but the rate is not a significant factor in peripheral perfusion. The color and temperature of the extremities are important for patients to observe. The remaining statements are true.

A patient with VTE is to be discharged on long-term warfarin (Coumadin) therapy and is taught about prevention and continuing treatment of VTE. The nurse determines that discharge teaching for the patient has been effective when the patient makes which statement? a. "I should expect that Coumadin will cause my stools to be somewhat black." b. "I should avoid all dark green and leafy vegetables while I am taking Coumadin." c. "Massaging my legs several times a day will help increase my venous circulation." d. "Swimming is a good activity to include in my exercise program to increase my circulation."

d. "Swimming is a good activity to include in my exercise program to increase my circulation." Exercise programs for patients recovering from VTE should emphasize swimming, which is particularly beneficial because of the gentle, even pressure of the water. Coumadin will not blacken stools. If this occurs, it could be a sign of gastrointestinal bleeding. Dark green and leafy vegetables have high amounts of vitamin K and should not be increased during Coumadin therapy, but they do not need to be restricted. The legs must not be massaged because of the risk for dislodging any clots that may be present.

The nurse in a primary care provider's office is assessing several patients today. Which patient is most at risk for a stroke? a. A 92-yr-old female patient who takes warfarin (Coumadin) for atrial fibrillation b. A 28-yr-old male patient who uses marijuana after chemotherapy to control nausea c. A 42-yr-old female patient who takes oral contraceptives and has migraine headaches d. A 72-yr-old male patient who has hypertension and diabetes mellitus and smokes tobacco

d. A 72-yr-old male patient who has hypertension and diabetes mellitus and smokes tobacco Stroke risk increases after 65 years of age. Strokes are more common in men. Hypertension is the single most important modifiable risk factor for stroke. Diabetes mellitus is a significant stroke risk factor, and smoking nearly doubles the risk of a stroke. Other risk factors include drug abuse (especially cocaine), high-dose oral contraception use, migraine headaches, and untreated heart disease such as atrial fibrillation

3. A patient comes to the emergency department immediately after experiencing numbness of the face and an inability to speak but while the patient awaits examination, the symptoms disappear and the patient requests discharge. Why should the nurse emphasize that it is important for the patient to be treated before leaving? a. The patient has probably experienced an asymptomatic lacunar stroke. b. The symptoms are likely to return and progress to worsening neurologic deficit in the next 24 hours. c. Neurologic deficits that are transient occur most often as a result of small hemorrhages that clot off. d. The patient has probably experienced a transient ischemic attack (TIA), which is a sign of progressive cerebrovascular disease.

d. A transient ischemic attack (TIA) is a temporary focal loss of neurologic function caused by ischemia of an area of the brain, usually lasting an hour or less. TIAs may be due to microemboli that temporarily block blood flow and are a warning of progressive cerebrovascular disease. Evaluation is necessary to determine the cause of the neurologic deficit and provide prophylactic treatment if possible.

16. What is a nursing intervention that is indicated for the patient with hemiplegia? a. The use of a footboard to prevent plantar flexion b. Immobilization of the affected arm against the chest with a sling c. Positioning the patient in bed with each joint lower than the joint proximal to it d. Having the patient perform passive range of motion (ROM) of the affected limb with the unaffected limb

d. Active range of motion (ROM) should be initiated on the unaffected side as soon as possible and passive ROM of the affected side should be started on the first day. Having the patient actively exercise the unaffected side provides the patient with active and passive ROM as needed. Use of footboards is controversial because they stimulate plantar flexion. The unaffected arm should be supported but immobilization may precipitate a painful shoulder-hand syndrome. The patient should be positioned with each joint higher than the joint proximal to it to prevent dependent edema

A 74-yr-old man who has right-sided extremity paralysis related to a thrombotic stroke develops constipation. Which action should the nurse take first? a. Assist the patient to the bathroom every 2 hours. b. Provide incontinence briefs to wear during the day. c. Administer a bisacodyl (Dulcolax) rectal suppository every day. d. Arrange for several servings per day of cooked fruits and vegetables

d. Arrange for several servings per day of cooked fruits and vegetables Patients after a stroke frequently have constipation. Dietary management includes the following: fluid intake of 2500 to 3000 mL daily, prune juice (120 mL) or stewed prunes daily, cooked fruit three times daily, cooked vegetables three times daily, and whole-grain cereal or bread three to five times daily. Patients with urinary incontinence should be assisted to the bathroom every 2 hours. Suppositories may be ordered for short-term management if the patient does not respond to increased fluid and fiber. Incontinence briefs are indicated as a short-term intervention for urinary incontinence

The nurse observes a student nurse assigned to initiate oral feedings for a 68-yr-old woman with an ischemic stroke. Which action by the student will require the nurse to intervene? a. Giving the patient 1 oz of water to swallow Incorrect b. Telling the patient to perform a chin tuck before swallowing c. Assisting the patient to sit in a chair before feeding the patient d. Assessing cranial nerves III, IV, and VI before attempting feeding

d. Assessing cranial nerves III, IV, and VI before attempting feeding The majority of patients after a stroke have dysphagia. The gag reflex and swallowing ability (cranial nerves IX and X) should be assessed before the first oral feeding. Cranial nerves III, IV, and VI are responsible for ocular movements. To assess swallowing ability, the nurse should elevate the head of the bed to an upright position (unless contraindicated) and give the patient a small amount of crushed ice or ice water to swallow. The patient should remain in a high Fowler's position, preferably in a chair with the head flexed forward, for the feeding and for 30 minutes following

A thoracic aortic aneurysm is found when a patient has a routine chest x-ray. The nurse anticipates that additional diagnostic testing to determine the size and structure of the aneurysm will include which test? a. Angiography b. Ultrasonography c. Echocardiography d. Computed tomography (CT) scan

d. Computed tomography (CT) scan A CT scan is the most accurate test to determine the length and diameter of the aneurysm and whether a thrombus is present. The other tests may also be used, but the CT scan yields the most descriptive results.

Which observation made by the nurse should indicate the presence of the complication of graft thrombosis after aortic aneurysm repair? a. Cardiac dysrhythmias or chest pain b. Absent bowel sounds, abdominal distention, or diarrhea c. Increased temperature and increased white blood cell count d. Decreased pulses and cool, painful extremities below the level of repair

d. Decreased pulses and cool, painful extremities below the level of repair Decreased or absent pulses in conjunction with cool, painful extremities below the level of repair indicate graft thrombosis. Dysrhythmias or chest pain indicates myocardial ischemia. Absent bowel sounds, abdominal distention, diarrhea, or bloody stools indicate bowel infarction. Increased temperature and white blood cells (WBCs), surgical site inflammation, or drainage indicates graft infection

A patient being admitted with a stroke has right-sided facial drooping and right-sided arm and leg paralysis. Which finding should the nurse expect? a. Impulsive behavior b. Right-sided neglect c. Hyperactive left-sided tendon reflexes d. Difficulty comprehending instructions

d. Difficulty comprehending instructions

Which aneurysm is uniform in shape and a circumferential dilation of the artery? a. False aneurysm b. Pseudoaneurysm c. Saccular aneurysm d. Fusiform aneurysm

d. Fusiform aneurysm The fusiform aneurysm is circumferential and relatively uniform in shape. The false aneurysm or pseudoaneurysm is not an aneurysm but a disruption of all the arterial wall layers with bleeding that is contained by surrounding anatomic structures. Saccular aneurysms are the pouchlike bulge of an artery.

Which indirect thrombin inhibitor is only administered subcutaneously and does not need routine coagulation tests? a. Warfarin (Coumadin) b. Unfractionated heparin (Heparin) c. Hirudin derivatives (bivalirudin [Angiomax]) d. Low-molecular-weight heparin (enoxaparin [Lovenox])

d. Low-molecular-weight heparin (enoxaparin [Lovenox]) Low-molecular-weight heparin (LMWH) (enoxaparin [Lovenox]) is only given subcutaneously and does not need routine coagulation testing. Unfractionated heparin is the only other indirect thrombin inhibitor option. It can be given subcutaneously or IV and therapeutic effects must be monitored with coagulation testing.

To help prevent embolization of the thrombus in a patient with acute VTE and severe edema and limb pain, what should the nurse teach the patient to do first? a. Dangle the feet over the edge of the bed q2-3hr. b. Ambulate around the bed three to four times a day. c. Keep the affected leg elevated above the level of the heart. d. Maintain bed rest until edema is relieved and anticoagulation is established.

d. Maintain bed rest until edema is relieved and anticoagulation is established. With acute VTE, prevention of emboli formation, decreased edema and pain can be achieved initially by bed rest and limiting movement of the involved extremity. Ambulation will be the next priority. Dangling the legs promotes venous stasis and further clot formation. Elevating the affected limb will promote venous return, but it does not prevent embolization.

The priority intervention in the emergency department for the patient with a stroke is a. intravenous fluid replacement b. administration of osmotic diuretics to reduce cerebral edema c. initiation of hypothermia to decrease the oxygen needs of the brain d. maintenance of respiratory function with a patent airway and oxygen administration

d. Maintenance of respiratory function with a patent airway and oxygen administration

20. A patient's wife asks the nurse why her husband did not receive the clot busting medication (tissue plasminogen activator [tPA]) she has been reading about. Her husband is diagnosed with a hemorrhagic stroke. What is the best response by the nurse to the patient's wife? a. "He didn't arrive within the timeframe for that therapy." b. "Not everyone is eligible for this drug. Has he had surgery lately?" c. "You should discuss the treatment of your husband with his doctor." d. "The medication you are talking about dissolves clots and could cause more bleeding in your husband's brain."

d. Recombinant tissue plasminogen activator (tPA) dissolves clots and increases the risk for bleeding. It is not used with hemorrhagic strokes. If the patient had a thrombotic or embolic stroke, the timeframe of 3 to 4.5 hours after onset of clinical signs of the stroke would be important as well as a history of surgery. The nurse should answer the question as accurately as possible and then encourage the wife to talk with the physician if she has further questions.

The female patient has been brought to the emergency department complaining of the most severe headache of her life. Which type of stroke should the nurse anticipate? a. TIA b. Embolic stroke c. Thrombotic stroke d. Subarachnoid hemorrhage

d. Subarachnoid hemorrhage Headache is common in a patient who has a subarachnoid hemorrhage or an intracerebral hemorrhage. A TIA is a transient loss of neurologic function usually without a headache. A headache may occur with an ischemic embolic stroke, but severe neurologic deficits are the initial symptoms. The ischemic thrombotic stroke manifestations progress in the first 72 hours as infarction and cerebral edema increase.

A patient with a left-brain stroke suddenly bursts into tears when family members visit. How should the nurse respond? a. Use a calm voice to ask the patient to stop the crying behavior. b. Explain to the family that depression is normal following a stroke. c. Have the family members leave the patient alone for a few minutes. d. Teach the family that emotional outbursts are common after strokes.

d. Teach the family that emotional outbursts are common after strokes.

12. Priority Decision: What is the priority intervention in the emergency department for the patient with a stroke? a. Intravenous fluid replacement b. Administration of osmotic diuretics to reduce cerebral edema c. Initiation of hypothermia to decrease the oxygen needs of the brain d. Maintenance of respiratory function with a patent airway and oxygen administration

d. The first priority in acute management of the patient with a stroke is preservation of life. Because the patient with a stroke may be unconscious or have a reduced gag reflex, it is most important to maintain a patent airway for the patient and provide oxygen if respiratory effort is impaired. IV fluid replacement, treatment with osmotic diuretics, and avoiding hyperthermia may be used for further treatment.

25. The nurse can assist the patient and family in coping with the long-term effects of a stroke by doing what? a. Informing family members that the patient will need assistance with almost all ADLs b. Explaining that the patient's prestroke behavior will return as improvement progresses c. Encouraging the patient and family members to seek assistance from family therapy or stroke support groups d. Helping the patient and family to understand the significance of residual stroke damage to promote problem solving and planning

d. The patient and family need accurate and complete information about the effects of the stroke to problemsolve and make plans for chronic care of the patient. It is uncommon for patients with major strokes to return completely to prestroke function, behaviors, and role and both the patient and family will mourn these losses. The patient's specific needs for care must be identified and rehabilitation efforts should be continued at home. Family therapy and support groups may be helpful for some patients and families.

A patient in the emergency department with sudden-onset right-sided weakness is diagnosed with an intracerebral hemorrhage. Which information about the patient is most important to communicate to the health care provider? a. The patient's speech is difficult to understand. b. The patient's blood pressure (BP) is 144/90 mm Hg. c. The patient takes a diuretic because of a history of hypertension. d. The patient has atrial fibrillation and takes warfarin (Coumadin).

d. The patient has atrial fibrillation and takes warfarin (Coumadin).

A patient arrives in the emergency department with hemiparesis and dysarthria that started 2 hours previously. Health records show a history of several transient ischemic attacks (TIAs). What should the nurse anticipate for this patient? a. Surgical endarterectomy b. Transluminal angioplasty c. Intravenous heparin drip administration d. Tissue plasminogen activator (tPa) infusion

d. Tissue plasminogen activator (tPa) infusion

Which intervention is most appropriate when communicating with a patient with aphasia after a stroke? a. Present several thoughts at once so that the patient can connect the ideas. b. Ask open-ended questions to provide the patient the opportunity to speak. c. Finish the patient's sentences to minimize frustration associated with slow speech. d. Use simple, short sentences accompanied by visual cues to enhance comprehension.

d. Use simple, short sentences accompanied by visual cues to enhance comprehension. When communicating with a patient with aphasia, the nurse should present one thought or idea at a time. Ask questions that can be answered with a "yes," "no," or simple word. Use visual cues and allow time for the individual to comprehend and respond to conversation

1. The nurse can assist the patient and the family in coping with the long term effects of a stroke by a. informing family members that the patient will need assistance with almost all ADLs b. explaining that the patient's prestroke behavior will return as improvement progresses c. encouraging the patient and family members to seek assistance from family therapy or stroke support groups d. helping the patient and family understand the significance of residual stroke damage to promote problem solving and planning

d. helping the patient and family understand the significance of residual stroke damage to promote problem solving and planning


Kaugnay na mga set ng pag-aaral

Chapter 44: Assessment and Management of Patients with Biliary Disorders

View Set

Finance Management Chapter 11 - FIN 780

View Set

Education 2130 Midterm: Chapter 1

View Set

IELTS Masterclass Unit 2 Conflicting Interests

View Set

Psych 354 Final Exam (chapters 1-14)

View Set